You are on page 1of 61
CHAPTER Fluid mechanics Inside d Pressure «+ Pressure in a fluid + Density of aliquid + Pascal's law + Archimedes’ principle Flow of fluids + Steady flow + Turbulent flow += Equation of continuity Bernoulli's theorem + Applications based on Bemoulli’s theorem Viscosity Stoke's law Surface tension + Surface energy + Angle of contact + Capillarity| ‘The term “fluid? refers to a substance which cannot withstand shear so, it has no definite shape of its own and it has the ability to flow. Thus, the term fluid applies iy both liquids and gases. In this chapter, we will study some important properties of fluids like pressure, viscosity, surface tension etc. 1) Pressure Normal force acting per unit area of a surface is called pressure. It is generally represented by . F Fig. 13.1 If surface area is A and normal force acting on the surface is F (shown in figure) pressure on the surface is p= Normalforce (F) Area (4) p= lim SF dF aso AA dA Iti clear from the above formula that, pressure acting on a surface depends uP? normal force acting on the surface and its area. Thus, force of same magnitude different pressure on different area that is for the same force if area is lesser the" pressure will be greater. If area is very small, then SI unit of pressure is Nm? or pascal (Pa). Pre: i a). Pressure is a scal intity and its dimensional formula is [ML“'T~*), heey ure in a fluid a fluid (either liquid or gas) is at rest, it se it exerts je perpendicular to any surface in contact with ach feyeontainer wall or a body immersed in the fluid, ie the fluid a8 a whole is at rest, the molecules or = that makes up the fluid are in motion. These paecules oF atoms collides continuously with walls of Sruainer. Pressure of a fluid is due to molecules Satding with the container walls, ‘he pressure applied by a fluid can be defined as the sxgtude of the normal force (applied by fluid) acting per voit surface area. ihe pressure is the same at all points ofa finite plane cerfce with area A, then — ene sshete, Fis the normal force on the surface. Unit of fluid pressure French scientist Pascal canributed many researches in the field of fluid pressure. Thus, in his honour SI unit of pressure is known as pascal hich is denoted by Pa, 1Pa =1Nm* One unit used practically in meterology is the bar which is equal to 10° Pa. 1 bar =10° Pa Atmospheric pressure (Patm) ‘tis pressure of the earth's atmosphere. This changes with ‘eather and elevation. Normal atmospheric pressure at sea 4evel(an average value) is 1.013x 10° Pa. Thus, Latm =1.013x 10° Pa Stier practical units of pressure are bar and torr (mm cig 1 atm = 1.01 bar = 760 torr "9" hid esr ats prendre any sue te id nate how tat sae vente Posies nou Sheen ft Soest ss 2 se i solute pressure and gauge pressure ®xcess pressure above atmospheric pressure 7 sally sig SUE pressure and the total pressure is calle “te pressure. Thus, - erie presure] je btessure re — atmosph wm Ne pressure is always greater than or equal to 2270 auge pressure con be negative also. Chapter 13: Fluid mechanics (597 Example 13.1 An open container has dimensions of 4.0m x 5.0m and height of 3.0 m. @ Find the weight of the air in the container at 20 °C. Gi) What is the weight of an equal volume of water? Also find pressure at the base of container due to this weight of water. (iit) What isthe total downward force an the base of the container due to air pressure of 1.0 atm? Take the densities of air and water as 1.2 kg/m and 10° ky/m® respectively. Sol, It is given that density of air py, =1.2kg/m? and density of water Pyaiee =10° kg/m! ) The volume of air in the container = (4.015.0(3.0)= 60m? ‘The mass of the air is mu. = py, -V, (1.2) (60) = 72 ky, man9 = (1219.8) = 705.6N eater = Pwater “Vwater 10° X6019.8) 9x 10° N Pressure at the base due to this weight w) _ 5.9 10° A 20 ai) The downward force on the base is (Air pressure) (surface area) 1.013 x 10°) (4.0% 5.0)=2.0% 10°N Note The above force has large magnitude. This force i enough to collapse the base. Why does it not collapse? Itis due to the upward force onthe underside of the base. iT we neglect the thickness of the base, this upward force is exactly equal in magnitude to downward force on the base and hence the net force due to air pressure Is Ze Density of a liquid Density (p) of a liquid or any substance is defined as the ‘mass per unit volume. ‘The weight of the air is Wie (Gi) The weight of water Wenee 2.95 x 10 Nin? Mass Volume Density, avo AV SI unit of density is kgm~* and its dimension is (ML? T°] and CGS unit is gec' with Igce™ = 10? kg m~3, Relative density (RD) In case of a liquid (or a substance), sometimes an another term relative density (RD) or specific gravity is defined. It is the ratio of density of the substance (or a liquid) to the density of water at 4°C. RD = Density of liquid or substance | Density of water at 4°C Hence, > 598) OBJECTIVE PHYSICS Vol. 1 Relative density is a pure ratio so, it has no units. It is also sometimes referred as specific gravity. Density of water at 4°C in CGS is 1 gem™. Therefore, numerically the RD and density of a substance in CGS are equal. In SI unit the density of water at 4°C is 1000 kgm”. Example 13.2 Relative density of an oil és 0.8. Find the absolute density of oil in CGS and SI units Sol, Density of oil (in CGS) = (RD) gem™ = 800 kgm” 18 gem? Density of a mixture of two or more liquids Case 1 Suppose two liquids of densities p, and p, having masses m, and m, are mixed together. Then the density of the mixture will be Total mass__ (m + ma) Total volume (V, + V2) (ny +2) (3-3) Pi Pa 22 Pi +P Case 2_Iftwo liquids of densities p, and p having volumes V, and Vz are mixed, then the density of the mixture is, If my then p= Total volume Vj+V, Vi +Vo Pi+P2 If fy then p= Pi Pa Example 83 Two liquids of densities p and 3p having volumes BV andV are mixed together. Find density of the mixture. Sol. Given, density of first liquid, p 3p Density of second liquid, p Volume of first liquid, Y, ‘Volume of second liquid, Vp =¥ c. Density of the mixture pV + pV, _px3V+3pxV _ 6p Y+V, aVeV 4 Effect of temperature on density When the temperature of a liquid is increased, the mass remains the same while the volume is increased and hence, the density of the liquid decreases (= pe 3) . Density after increase of temperature by A@ _ p’ Tnitial volume of density Pp v v Vedv V+VyA0 Thus, or & P Here, and a0 Therefore, Effect of pressure on density ‘As pressure is increased, volume decreases and hence density will increase. Thus, p & = Density after increase of pressure _ p’ Tnitial density a or == pe _& B Here, dp = change in pressure, and B= bulk modulus of elasticity of the liquid Variation of fluid pressure with depth Consider a fluid of density p is kept at rest; in a cylindrical vessel of height has shown in figure. Cylindrical vessel Fig. 13.2 Pressure applied by liquid at the bottom of the cont [p= hog] d From the above formula, we can He Eo r la, we can say that presst by a liquid column is proportional to height of liquid col™"™ If the container is open, then atmospheric pressure also work on the top surface, so total pressure of th? bottom will be equal to pat [Pr Po (atmospheric pressure) an a. (, We can also find the difference of pressure ee ‘any two points in the liquid column, Fig. 13.3 consider two points A and Bin a liquid column situated at jy and hy respectively below free surface of liquid. prasure due to liquid column at point Ap, = hypg pressure due to liquid column at point B, py Hence, the pressure difference depends on the vertical height ( ie., distance between point A and point B), density of the fluid and the acceleration due to gravity. According to Eq. (i) pressure increases linearly with depth, ifpand gare uniform. Agraph between p and h is shown below. " ‘p= ro + poh - h Fig. 13.4 Graph between pressure and height ‘mportant points related with fluid pressure Inportant points related with fluid pressure are given below i) Ata point in the liquid column, the pressure applied on it is same in all directions. (i) Ina liquid, pressure will be same at all points at the ay Same level = i) The pressure exerted by a liquid depends only on the height of fluid column and is independent of the shape of the containing vessel. wd wn Ox Fig. 13.5 The three vessels A, B and C containing different amounts of liquids Thhy = hy = he, then pa = Pa = PC : id mechanics | 599 Chapter 13 : Fluid me & (iv) Consider following shapes of vessels @ Fig. 13.6 Pressure at the base of each vessel Px =Py =P: =Po +pghbutw, #w, #w, where p = density of liquid in each vessel h = height of liquid in each vessel po = atmospheric pressure (0) In the figure, a block of mass ‘floats over a fluid surface fo Bleck of If p = density of the liquid A= area of the block Pressure at the base of the vessel in Example BA Find the pressure exerted below a column of water, open to the atmosphere, at depth 10m Gi 30 m (Given, density of water = 1x 10°kgm~, g =10ms*) Sol () Pressure at a depth of 10 m P= pq + Pgh =1.013 x 10° Pa + (1x 10° kgm™®)0ms~? 40m) =1.013 x 10° Pa + 1x 10°Pa =2.013 x 10°Pa Gi) Pressure at a depth of 30 m P= Pat Pgh 1,013 x 10° Pa + (1 10° kgm™\LOms"2\30m) =1.013 « 10°Pa +3 10°Pa = 4.013 x 10° Pa=4 atm Example 13.5 For the arrangement shown in the figure, what is the density of oil? oil ™ ~ 800) OBJECTIVE PHYSICS Vol 7 Sol. According to the figure, for the equilibrium of liquid Net downward force = Net upward force Po+Pw 9! = Po + Pai + d)g Let Fe the force exerted by side walls on the wy, (upwards), Then for equilibrium of water (EBD) "= Net upward force= net downward force Bul _1000% (135) 3 F+R=Fj tw a i 100% 135) 916 kgm or =F +d (1354 123) “916 Kem a F =F, +w~Fy=303 +10~109 = 211 N(upwar. Example 13.6 A curved glass vessel full of water upto a height * Poa Neoware) of 10 cm has a bottom of arealO em:, top of area 30 em and volume 1 L. (@) Find che force exerted by the water on the bottom, (ii) Find the resultant force exerued by the sides of the glass on the (Gt) Ifthe glass vessel is covered by a jar and the air inside che jar is completely pumped ot, wha lb thease pr (0 ered (iv) If a glass vessel of different shape is used provided the height, the bottom area and the volume are unchanged, will the answers 0 pares (a) and (b) change. (Take, g=10ms"*, density of water =10° kgm"? and armespheric pressure =1.01%10° Nn“) Sol, (i) Force exerted by the water on the bottom F=(0 + pamnA, Here, py p= density of water 0° kgm 10ms*, h=10em=0.1m area of base = 10 em? = 10° m? 30 em? = (Top) 2 € he Volum 1Ocm'= 4, Bottom) Substituting these values in Eq. (, we get F,= (1.01 105 + 10x 10x 0.1) 107 or F,=102N (downwards) (ii) Force exerted by atmosphere on water Fy =(polA2 Here, A; = area of top = 30 cm? = 3x 107 m? F, = (1.01 10°)3x 107) = 303 N (downwards) Force exerted by bottom on the water Fy=—F, or Fy=102N (upwards) Weight of water, w = (volume)(density)(9) =(107°\10° X10) = 10N (downwards) smospheric pressure = 1,01 10° Na If the air inside the jar is completely pumped out, Fi = (oh), (asp =0) = (10°10X0.1)110) = LN downwards) F=0 Fy=1N (upwards) F=r,+w-Fy =0+10-1 =9N (upwards) Gv) No, the answer will remain the same. Because the answers depend upon ppp gh Ay and Ap. Pascal's law It states that, “pressure applied to an enclosed fluid i transmitted undiminished to every portion of the fluid and the walls of the containing vessel”. Fig. 13.8 Pressure applied to an enclosed fluid As an example in which a flask fitted with a piston © filled with a liquid. Let an external force F is be apple’ ‘on the piston. If the cross-sectional area of the pisto” #5, the pressure just below the piston is increased by £/: Pascal's law the pressure applied by piston will be transmitted equally at all points in the flask. If We some holes in the flask then the liquid from all the will emerge out with same intensity. make oles A hi R A A Fig. 13.9 Hydraulic tif, piston with small cross-section area fathe surface of a liquid such as oil. = FV/A is transmitted through the fuger piston of area Ay. It pushes lar f;, The applied pressure is the same in both eylinders, so A, exerts a force F; ‘The applied pressure connecting pipe toa ger piston with force oe a A A, A Now, since Ap >> A,, therefore, F, >> F;. Thus, hydraulic litis a force multiplying device with a multiplication factor equal to the ratio of the areas of the two pistons. Dentist’s chairs, car lifts and jacks, hydraulic elevators and Indraulic brakes all use this principle. Gromple 12.7 Figure shows a hydraulic press with the larger sivan of diameter 35 cm at a height of 1.5 m relative to the smaller piston of diameter 10 em. The mass on the smaller fidonis 20 hg. What isthe force exerted on the load by the leger piston? The density of oil in the press is 750 kgm” - Take, g=9.8 ms"?) olla ae [i 20x98 Nae? “LPressure on. the smaller piston = Nev x (5x 107)? | nx (17.5% 10" The difference between the two pressures =p Pressureon the larger piston = hae h=1sm 7 p= 750 kgm? . Th 20x98 _ : ae Gxlo? ae TSHT 5x 150% 98 “hich gives, F = 1.3% 10° nd has been ‘Atmospheric pressure is common to both pistons a itnoreg Chapter 13 : Fluid mechanies (on Example 13.8 Two pistons of a hydraulic machine have diameters 20 cm and 2 cm. Find the force exerted on the larger piston when 50 kg wt is placed on the smaller piston. When the smaller piston moves is through 50 cm, by what distance the other piston moves out? Sol, For smaller piston, area A, = mx (em)? A, =x (0cm)* Force exerted on the larger piston, For larger piston, area A em FywAt 5 MOM? sox 0.8 A xl emy? =100x 50x 9.6=5x10'N This is the force exerted on the larger piston. The liquids are considered incompressible. Therefore, volume covered by ‘movernent of smaller piston inwards equal tothe outward movement of lager piston. x(1cm)? a taste te( deaths 1 Lx 5m =0.05 cm 00 So, the distance moved out by the larger piston is 0.05 em. Measurement of pressure Pressure can be measured by using following two devices 1. Barometer It is a device used to measure atmospheric pressure. In principle, any liquid can be used to fill the barometer, but mercury is the substance of choice because its great density makes possible an instrument of reasonable size, A barometer is an inverted evacuated tube, put over a mercury volume. Outside pressure pushes mercury into tube till the weight of liquid column equalises the force due to external pressure. In given diagram in equilibrium P= Pe Here, _p, = atmospheric pressure (py) and p= 0+pgh =pgh Fig, 13.10 Barometer ~ 802) OBJECTIVE PHYSICS Vol. 1 Also, p = density of mercury Po =pgh Thus, the mercury barometer reads the atmospheric pressure (pq) directly from the height of the mercury column. 2g. if the height of mercury in a barometer is 760 mim, then atmospheric pressure will be, Po =pgh = (13.6x 10°)(9.8(0.760)=1.01x 10° Nm? Example 13.9 What will be the length of mercury colurnn in a barometer tube, when the atmospheric pressure is 76 cm of ‘mercury and the tube is inclined at an angle of 30° with the horizontal direction? Sol, Here, h= 76 cm, 8 =30° If is the length of mercury column is the barometer tube then ioe T lea > 1=2x 16 =152em 2, Manometer It is a device used to measure the pressure of a gas inside ‘a container. Fig. 13.11 Manometer The U-shaped tube often contains mercury. Let 1 and 2 are points on same horizontal level then, PL = Pa Here, p; = pressure of the gas in the container (p) and p= atmospheric pressure (pg) + pgh P= Po +pgh This can also be written as P— Po = gauge pressure = pgh Here, p is the density of the liquid used in U-tube. Thus, by measuring h we can find absolute (or gauge) pressure in the vessel. Example 13.10, A manometer tube contains a liquid of density 4x 10° kgm”, When connected to a vessel containing a gas, the liquid level in the other arm of the tube is higher by 20 cm. ‘When connected to another sample of enclosed gas, the liquid level in the other arm of the manometer tube falls 8 cm below the liquid level in the first arm, Which of the two samples exerts more pressure and by what amount? Sol. For sample 1 Difference in level of liquids, hy =20 er Pressure of the gas in the containing, A =p, + pg, r For sample 2 In this case, level of the liquid in the left arm is higher than that in the tight arm by 8 cm. Atmospheric pressure p, is greater than the pressure exerted by the sample = Pa=P2* Poh, = Pr=Pa—Pahy ‘Comparing equations (j) and (i), it is clear that — > p). Therefore the gas in sample 1 exerts greater pressure than that in sample 2 ‘The difference in the two pressures is A~P2 =(P, + pahy)— (Pp, Paha) =pagth, + ha) =pg (28cm) =x 10° kgm”) x (9.8msX0.28m) =8.23x 10° Pa= 8 kPa Archimedes' principle If a heavy object is immersed in water, it seems to weig! less than when it is in air. This is because the water © ‘net upward normal force called buoyant fore. The Archimedes principle gives the magnitude of buoy" force on a body. It states that, “A body wholly or parti submerged in a fluid experiences an upward force Wb" is equal to the weight of the displaced fluid.” magnitude of buoyant force (F) whi 8 hs is given by, ‘which is also i 3 F=vna] Fig. 13.12 Net upward force on immersed object Y, = immersed volume of solid pr = density of liquid eo g = acceleration due to gravity ‘he upthrust act vertically upwards through the centre of gasity of displaced fluid. sie Apparent weight of a body inside a liquid iabody is completely immersed in a liquid its effective wright gets decreased. The decrease in its weight is equal ‘othe upthrust on the body. ‘koe, Apparent weight of body = Wags = Wresas—upthrust Wage =VPs9-Verg Here, V =total volume of the body Ps =density of body wip, = density of liquid Tus, | Wig =Valp.s — Px) Pests may now arse ®s< pz, then in this condition the upthrust “plied by the liquid will be greater than the weight tithe body. That means if the body is completely fatetsed in liquid it will experience a net upward s. When released the body comes up to the fluid “Stface till the upthrust becomes equal to the weight ~ Chapter 13 : Fluid mechanics (os of the body, at this point the body floats partially immersed in the fluid. Fig. 13.13 (i) fps =p, s then W,», = O in this condition upthrust of the liquid balances the weight of the body. The body floats completely submerged just below the surface of the fluid. Fig, 13.14 ii) If ps > pz then in this condition the upthrust of the liquid is less than the weight of the liquid. ie, it is not sufficient to balance the weight of the body, so the body sinks. Fig, 13.15 Example 13.1 An omament weighing 50 g in air weights only 46 g is water. Assuming that some copper is mixed with gold to prepare the ornament. Find the amount of copper in it. Specific gravity of gold is 20 and that of copper is 10. Sol, Let m be the mass of the copper in ornament, ‘Then mass of gold in i is (50 ~m). Te s [ and volume of gold V, = 50=” 20 ‘Volume of copper Volume = Mass Density ‘When immersed in water (p,, =1 gem"2) Decrease in weight = upthrust (60-4619 = +¥4)pu9 m , 50-m or aeMy = 10 * a9 °F 80=2m + 50m m=309 si) OBJECTIVE PHYSICS Vol. 7 7 Example TB12_ Density of ice is 900 kgm”. A piece of ice is floating in water (of density 1000 keg m™*). Find the fraction of volume of the piece of ice outside the water. sat Let V be the total volume and Vj the volume of ice Piece immersed in water. For equilibrium of ice piece, bd) 1p, —density of ice Here, py =density of water = 1000 kgm? and ‘Substituting in Eq. (i), we get M8» M09 V py 1000 ie., the fraction of volume outside the water, floating in a glass vessel filled in the vessel change fe1-09 Example 13.13. A piece of ice is with water. How will the level of water i when the ice melts? Sol, Let me the mass of ice piece floating in water. In equilibrium, weight of ice piece = upthrust v o mg=V. Pug oF Pw Here, Vj is the volume of ice piece immersed in water. When the ice melts, let V be the volume of water formed by mrass of ice. Then, v-2 enlii) Pe From Eqs. (i) and (i, we see that, V =V Hence, the level will not change- Example 13.14 A piece of ice having a stone frozen in it floats in a glass vessel filed with water. How will the level of water in the vessel change when the ice melts? Sol, Let, m= mass of ice, my = mass of stone, ps density of stone, and py = density of water. In equilibrium, when the piece of ice floats in water, weight of (ice + stone) = upthrust (on, + m)9=¥; Pug y= fl Pe Pw : Here, V;= Volume of ice immersed When the ice melts, m mass of ice converts into water and stone of mass m is completely submerged. ‘Volume of water formed by m, mass of ice, mH ‘Volume of stone (which is also equal to the volume of water displaced) Vy =72 Since, ps >Pu Ps Therefore, Y+V2) 12 Dimensional formula for pressure is fe) (MUT?) (b) (MET) (6) MUT~?] (6) [PUT] {1 Pins and nails are made to have pointed end because (@) transmits very small pressure {0 it transmits a large pressure (0) it provide a large area {6 None of the above ‘4. The two thigh bones each of cross-sectional area 15cm? suppor the upper part of a person of mass 70 kg. The pressure sustained by these thigh bones is (a) 2.5%10° Nm? (b) 1.33% 10° Nm? (©) 466x108 108 Nm? (¢) 2.93x10° Nm? ‘5. Which of the following is correct. (@) Gauge pressure = absolute pressure + atmospheric pressure (0) Absolute pressure = gauge pressure atmospheric pressure (©) Gauge pressure = absolute pressure ~ atmospheric pressvrh ure — gauge pressure (@ Absolute pressure = almospherc press 6 Ata depth of 500 m in an ocean, what is the absolute pressure? Given that the density of sea water 4103x10? kg m? and g = 10ms * (9) 40.a1m (o) 52atm © 32am (@) 62am 7. Four vessels 8, C and D have different shapes anc hold is correct. diferent amount of water. Which of the following (o) <<< (Fea Fer Fe mPa Chapter 13 : Fluid mechanics (60s F P CO Col To+mg T+mg Solving Eg. (iH) and (i, we get T=T (: o 3 jp having their volumes in the 8. Two liquids of densities 2p and, fe will be ratio 3:2 are mixed together. Density of the mixtur @ ® ws 8. 42 o® o*# two different liquids in static 7m and oil of unknown density ‘= 15mm, Density of the oil is 9. The U-tube in figure contains ‘equilibrium, water in the right ar peinthe left, If/ = 185mm anda oi (@) 1000 kg m? (©) 920kg m? (©) 895kg m? (@) 900 kg m? 10. Increase in pressure at one point of the enclosed liquid in ‘equilibrium at rest is transmitted equally to all other points. ‘This is as per (@) impulse (0) Pascal's law () conservation of momentum (@) None of the above 14. In avehicle lifter the enclosed gas exerts a force F on a small piston having a diameter of 8 cm. This pressure is iransmitted to a second piston of diameter 24 om. If the mass of the vehicle to be lifted is 1400 kg then value of Fis, (@) 1200N (b) 1800N (c) 1600N (d) 700N 42. What will be the length of mercury column in a barometer tube when the atmospheric pressure is x cm of mercury and, the tube is inclined al an angle ¢with the vertical direction? Ching ene Ome le sat) OBJECTIVE PHYSICS Vol. 1 cavities in the casting? Density of ifon is 7.87 gom-* 13. A.boat floating in a water tank is carrying a number of stones. dams? and densly of water = 10? kam Ifthe stones were unloaded into water, then the wate level (Toko. 9 = 9 ty) oom? (@) increases @ atem (d) 0.14m* (©) remains same (9 0.12mi « (0) decreases 415. A body floats in water with 40% ofits volume CUtsice Wate, (d) data is insufficient ‘When the same body floats in an oll 60% of its volume 14, An iron casting containing a number of cavities weights remains outside oil. The aos density of oilis {6000 N in air and 4000 N in water. What isthe volume of the @ 09 ) 12 @18 Properties of streamline are given below 2 i (i In streamline flow, no two streamlines can cross Flow of fluids cach other, If they do 50, the particles of the liquid St the point of intersection will have two different Till now we have studied about the fluids at rest. In this, directions for their flow, which will decrease the section we will study fluids in motion. When a fluid is in steady nature of the liquid flow. flow, its motion can either be smooth or irregular (Gi) The greater is the crowding of streamlines at a place depending on its velocity of flow. greater is the velocity of the liquid particles at that place and vice-versa. ii) Streamline flow is possible only if the liquid velocity Steady flow Goce not exceed a limiting value called critical Consider a liquid flowing in a tube as shown in the figure velocity. given below. If the velocity of fluid particles at any point _ iv) The bundle of stream lines forming a tubular region does not vary with time, the flow is said to be steady. is called a tube of flow. Steady flow is also called streamlined or laminar flow. The velocity at different points may be different. Hence, in the figure, Fig. 13.19 Tube of flow formed by streamlines Turbulent flow Fig. 13.18 Steady flow of liquid through a tube In rivers and canals where speed of water is quite high o" the boundary surfaces cause abrupt changes in velocity. we can say that velocity of fluid particles at different the flow, then the flow becomes irregular. Such flow of points 1, 2 and 3 remains same with time. liquid is known as turbulent flow. , Vj, = constant, V; = constant, V3 = constant. Thus, the flow of fluid in which velocity of all particles but Vy) #V2 #V3 oan. a given point is not same and the motion . the luid becomes irregular or disordered. This is calle Streamlines : turbulent flow. ‘The path followed by a fluid particle in steady flow is A few examples of turbulent flow are called streamline. Velocity of fluid particle at any point (i) ajet of air striking a fi Sf the streamline is along tangent to the curve at that hae (Gi) the smoke rising from a burning stock of wood oint. (i 7 (ii) when a fast flowing streames encounters r0ckS: foamy whirlpool like regions are formed. nolds number ber (R,)is a dimensionless number, whose is fs an idea whether the flow would be laminar valde ed or tarbulent “ Inertial force ©” Viscon lds 0 force presents the ratio of inertial force of moving fous force offered by the fluid, ion for R, is wuss Re Fe uid t0 ¥i8C rhe express p= density of fluid, v= speed of fluid D = diameter of tube t= viscosity of fluid (later on it is diseussed in detail) « For R, < 1000, flow is streamline or laminar. «For R, —> 1000 to 2000, flow is unsteady. « For R, > 2000, flow is turbulent. romple 13.16 Water is flowing in a pipe of diameter 6 em vith an average velocity 7.5 cms”. s~' and its density is 10! gm. What is the nature of low? Given coefficient of viseosity of water is 10"? kgm" s*. Sol Reynolds number for the given situation is given as Here, Cocfticient of viscosity, m = 10 kgm’ Average velocity of water, v = 75 cms” Diameter of pipe, D = 0.075 = 6cm Hence, p= 10° x 0075 006 an =10° x 0,0045 = 4500 R, > 2000 Therefoie, the flow is turbulent ®quation of continuity y Wes an incompressible and non-viscous liquid flows in a craatlined motion through a tube of non-uniform Pr~ Pr =09(h,— hy) Example 13.18 Calculate the rate of flow of glycerine of density 1.25% 10° kgm through the conical section of a horizontal pipe, if the radii of its ends are 0.1 m and 0.04 m ‘and the pressure drop across its length is 10 Nm~* Sol, According to the question, we draw the following diagram. Ave Aa Pa From continuity equation, Ay =A ce vy A om? -(2) (2) From Bernoulli's equation, A+ 5 PH pat pot tashot vp op = 21 - pa) p / _ | . 125% 10> Solving Eqs. i) and (i), we get : ©0128 ms * Rate of volume flow through the tube Q= Age, =tarFig wn (0.04)*0.128) = 6.43% 104 ms pol lications based on pernoulli's theorem applications based on Bernoulli's theorem are given below ,Venturimeter shows a venturimeter used to measure flow speed fevige. We apply Bernoulli’s equation to the wide (pint 1) and narrow (point 2) parts of a horizontal pipe, 4 1 with = hay Pi +5 PUT = P2 +5 pv} Fig, 13.22 Venturimeter From the continuity equation, v, = A141 Aa substituting and rearranging, we get 12 (A? : ~p, =X pv? (40-1 eli) Pi-P2= 3 Pri (2 Because Ay is greater than Az, v2 is greater than v, and hence the pressure p, is less than p,. A net force to the ‘ight accelerates the fluid as it enters the narrow part of the tube (called throat) and a net force to the left slows as itleaves. The pressure difference is also equal to pghy here his the difference in liquid level in the two limbs ‘famanometer tube. Substituting in Eq. (i), we get Ne he derstanding of tur effect can be used to give a qualitative ofan air plane wing andthe path ofa pitchers curveball An ne wings designe, so that air moves faster ove the top ofthe To thatt does uncer the wing, thus. making te ar pressure lessor than undern is diference in pressure results in a net an undemeath. This 6 ing force on the wing. —~ Z Chapter 13: Fluid mechanics (609 Example 13.19 The flow of blood in a large artery of an anesthetised dog is diverted through a venturimeter. The wider part of the meter has a cross-sectional area equal to that of the artery, A=16mm?. The narrower part has an area a = 8mm The pressure drop in the artery is 24 Pa. What is the speed of the blood in the artery? Sol. Bernoulli's equation for the horizontal flow of blood is ee ns tout ap + tout By equation of continuity, Ay, av, aoe: Here, —— pz (blood) 2. Speed of efflux The outflow of a fluid is called efflux and the speed of the liquid coming out is called speed of efflux. Consider a closed vessel filled with a liquid upto height Hf and a small hole is made in the wall of the vessel at a depth h below the surface of liquid. Then, the speed of efflux is, Fig. 13.23 Liquid coming out from the orifice of a tank From the above formula it is clear that “The velocity of efflux of a liquid issuing out of an orifice is the same as it would attain, if allowed to fall freely through the vertical height between the liquid surface and orifice.” ‘The above statement is also known as Torricelli’s theorem. Range The range of liquid (R) i.e., the horizontal distance covered by liquid coming out of the hole is given by . (rear =~ 610) OBJECTIVE PHYSICS Vol. 1 Ris maximum at h Fan Ray = 2 R? = 4h -h?) aR? For R to be _—— ‘or e maximum, dh o or H-2h=0 or h= 2 2 That iy Ris maximum at h = and > Rove H ie., the maximum horizontal distance covered by liquid coming out of a hole is equal to the height of the liquid column. ‘+ Time taken to empty a tank is given by the formula a 9 Example 1320 If the water emerge from an orifice ina tank in which the gauge pressure is 4x 10° Nm~ before the flow starts, then what will be the velocity of the water emerging out? Take density of water is 1000 kg m~. Sol, Here, p= 4x 10° Nm“ and p =1000kg m~, 9 =10ms? p__4x10° .ps ha B = 4x10" rel Rete pg 100010 A fax 10x 410° Velocity of efflux, v= 29h Example 1321 A cylindrical vessel of 90 cm height is kept filled with water upto the rim. It has four holes 1, 2, 3,4 which are respectively at heights of 20 cm, 45 cm and 50 em from the horizontal floor PQ. Through which of the holes water is falling at the maximum horizontal distance? ~ Sol, Different holes ofthe cylindrical vessel are shown in comes out from different holes with figure. Water comes out i speeds. Therefore range wil be different corresponding different holes. Ba when ‘This height corresponds to hole 3. Example 1322 A tank is filled with a liquid upto a height H, A small hole is made at the bottom of this tank. Let be the time taken to empty first half of the tank and t the time taken 10 empty rest half of the tank. Then find the value of Sol, As we know that, the time in which level of liquid in a tank falls from H, to Hy is t 4 Emm ava where, A= area of cross-section of the tank, a =area of hole It is given that H =H and Hy a 4 2 (vn (2) al Similarly time taken to empty the rest half of the tank nod 4 FE oi aie ale From Eqs. (i) and (i), we get or Acosta cs is Note From here we see thatt, 2000 flow is turbulent oer i (@ Allare incorrect, (© 45s" @ 15m ae 3. An incompressible mvs liquid is lowing through 8. The velocity of efflux of a liquid through an orifice in the horizontal pipe as bottom of a tank does not depend upon shown in figure. The ; (@) density of tquic value of speed vis ois EG (0) height of the quid column above orifice (@) 1.1ms* or (©) acceleration due to gravity 21 ms" ty (d) None of the above (2 3.1 ms) 9. The lift of an aeroplane is based on (0) 8:1 ms (@) Tortcel’s theorem (©) Bemoull's principle 4 Bernoul’s theorem is a consequence of (©) law of gravitation (@) continuity equation {8 conservation of mess 10. A tank is filed to a height H. The f wate a . fi io range of water coming out Cone ae Of @ hole which is a dep H/s from the surface of water level (9) conservation of angular momentum or Ga 5. Waters in streamtine flows along a horizontal pipe with oF w» "omuniform cross-section, Ata point in the pipe where the 8rea of cross-section is 10cm”, the velocity of water is (© VaH 3H @ > ‘ims and the pressure is 2000 Pa, 4) Viscosity Te characteristic of fluid by virtue of which relative “canesteer ferent layers is opposed is known as Visca ty is i it due to ag internal friction in a fluid it occurs = tees of molecules of one layer into another. Viscous Fig. 13.25 ‘othe ehBe8es the motion of one portion of a fluid relative The bottom plate is stationary and the top plate moves ig EF. The simplest example of viscous flow is with constant velocity v. The fluid in contact with each °f a fluid between two parallel plates. surface has same velocity as that of the surface. The flow a ~ G12) OBJECTIVE PHYSICS Vol. 1 speeds of intermediate layers of fluid increase uniformly from bottom to top, as shown by arrows. So, the fluid layers slide over one another According to Newton, the frictional force F (or viscous force) between two layers depends upon the following factors, (i) Force Fis directly proportional to the area (A) of the layers in contact, ie., FoA i) Force Fis directly proportional to the velocity gradient (2) between the layers. Combining these 9 * two, we have FaA® or nate dy dy Here, nis constant of proportionality and is called coefficient of viscosity. Its value depends on the nature of the fluid. The negative sign in the above equation shows that the direction of viscous force F is opposite to the direction of relative velocity of the layer. The SI unit of nis Nsm™. It is also called decapoise or pascal second. Thus, Dimensions of n are[ML"'T~'} Coefficient of viscosity of water at 10°C is = 1.3x 10 Nsm™. Experiments show that coefficient of viscosity of a liquid decreases as its temperature rises. 1 decapoise = 1 Nsm~ =1 Pa-s = 10 poise Example 13.23 A plate of area 2m* is made to move horizontally with a speed of 2 ms~' by applying a horizontal tangential force over the free surface of a liquid. The depth of the liquid is\ m and the liquid in contact with the bed is stationary. Coefficient of viscosity of liquid is 0.01 poise. Find the tangential force needed to move the plate. Sol. Velocity gradient 1 From Newton's law of viscous force, IF [=n 22 = (01x 10°')(2)(2)= 4x10 N ay ‘So, to keep the plate moving a force of 4 x 10° Nmust be applied. Flow of liquid through a cylindrical pipe Figure shows the flow speed profile for laminar flow of a viscous fluid in a long cylindrical pipe. fest along the axis and zero -The speed is greatest along 0 at the ping 8 Fig, 12.24 A viscous fluid flowing in along eslindricel pig The flow speed vat a distance rfrom the axis of a pipe radius Ris, Pi—P2 (2,2) anL. where, p and p2 are the pressure at the two ends of a pipe with length L. The flow is always in the direetion of decreasing pressure. ‘At r = R (along the walls) v = 0, ‘The velocity of viscous fluid along the wall of the pipe is zero. and At r = O (along the axis) (py — pa)R? anL The velocity of viscous fluid flowing along a pipe is maximum along its axis. Volume flow rate(o or *): Poiseuille equation To find the total volume flow rate through the pipe, we consider a ring with inner radius r, outer radius r + dr and cross-sectional area dA = 2nr dr. The volume flow rate through this element is v dA. The total volume flow rate is found by integrating from r = 0 to The result is, The relation was first derived by Poiseuille and is called Poiseuille’s equation. ‘Some important points related to Poiseuille equation are given below (@ Poiseuille's equation can also be written as, =Pi= Pe _ Ap _ nb (=) x Hee, X= mR* ‘This equation can be compared with the curren equation through a resistance, i.e. i Here, AV =potential difference and > jor current flow through a resist ance, potential difference isa requirement similarly for fey fiquid through « pipe pressure diffe, 4 Problems of series and parallel comby ‘ination of pipes ner as is done in The only difference is, replaced by the can be solved in the ‘ase of an electrical circu {a) Potential difference (AV) pressure difference (Ap), {b) The electrical resistance R ( me) aR* is replaced by volume io = £)is replaced by Pow rextance x (0) The electrical current flow rate Q or dV dt. Combination of capillary (thin) tubes {Series combination (a) The rate of flow (Q) will be same. (b) Pressure differece across each tube is different. cy 7 (_9=2 1 = pe 1 Fig, 13.26 By analogy, total flow resistance is, x, +x, = 0,41) Re Also, py ~ pz = QXy Po ~ py = QXp and py ~ ps = OX. (i) Parallel combination (a) The pressure difference across each tube is same. () The rate of flow across each tube is different. Flow resistance of tubes are x, = SL andx, = OnE, mR} n ne ta =o Um 4 Fig 13.27 : lectrical analogy, total flow resistance of parallel tubes ven, a Menby, todd 5 OK = OaKe Xa XX PL P2=Q, +Q:)Xeq — Chapter 13 : Fluid mechanics (os lowing examples illustrates above discussion. Example 1324 Water is flowing through a horizontal tube 8 cm in diameter and 4 kom in length at the rate of 20 litre/s. Assuming only viscous resistance. Find the pressure required to ‘maintain the flow in terms of mercury column. (Coefficient of viscosity of water is 0.001 Pas) is Sol Here, 2r = 8 em = 008 m oF r= 0.04mn,1= 4 km = 4000 m, = 20 litre/s = 20 x 10°? m?s~!, n = 0.001 Pa-s, p As, Va Bort wal Bn mrt = 8x 20% 10") 0.001 x 4000 22 7 22). 0.04 ( 7) : Height of mercury column for pressure difference p will be, ha 2 = 7.954 x 108] 79 (13.610) 9.8 Example 13.25 A liquid is flowing through horizontal pipes as shown in figure. = 7.954104 Pa 0.5968m = 59.68 cm Length of different pipes has the following ratio Lag = gy = Ew Et 2 Sima, radi of diferent pipes has the rato, Ray # Rep = Rey = Ra 2 Pressure at A is2pp and pressure at D is. The volume flow rate through the pipe AB is Q. Find, Po Le {a volume low rts through EF eed GE 1 pressure at E and F So) The eqivalen electrical cru can be dawn asunder xek, & lox Tre Xan: Xeo:Xep iXon 8:86:16: oo mM) OBJECTIVE PHYSICS Vol. 1 (a) As the eurent is distributed in the inverse tio of the Terminal velocity (vr) resistance in gral) The Ql be dstibated nthe smal phere falling fom rest through a tpg id ‘Thus, volume flow rate through EF will be 2 ‘and that column of viscous fli 7 from GH wil be 1 Q » (os Jae veh stn vera dane) Fig. 13.29 Force ating on a small sphere falling through a liquid Now, let be the pressure at E, then Bx 1p 2p - a= 8QX = =I cting on the sphere are, aaa The forces acting on the spl 1x8 {i) weight w of the sphere acting vertically downwards A -(2 eee Js =1537, (i) upthrust buoyant force F, acting vertically upwards (iii) viscous force F, acting vertically upwards, ive, ina Similarly, if p, be the pressure at F, then . a Sirecion opposite to veloety of the sphere 8x17 ‘tially, F,=0 co Pa=Pot ag Po of = 14TH Initially, [as v =0) and w>F, and the sphere accelerates downwards. As the velocity of the sphere increases, F, increases. Eventually a stage in 5) Stoke's law ee o After this net force on the sphere is zero and it moves ‘When an object moves through a fluid, it experiences a downwards with a constant velocity called terminal . viscous force which acts in opposite direction of its velocity (vp). velocity. i It can be defined as the maximum constant velocity i acquired by a body while falling through a viscous medium, ‘Substituting proper values in Eq. (i) we have, 3 4 i) 39 tog + Gen ca Fig. 13.28 An object is moving through a fluid Here, ‘The formula for the viscous force on a sphere was first i oo of sphere, o = density of fluid derived by the English physicist G. Stokes in 1843. " ficient of viscosity of fluid ‘According to him, a spherical object of radius rmoving at From Eq. (i), we get velocity v experiences a viscous force given by ip [F= 6m] (n= coefficient of viscosity) oc This law is called Stoke’ ng the variation of the velocity v ofthe sphere wth ol time Fig. 13.30 Variation of velocity ofa sphere fal through a liquid, with time sd pote from the bove expression ve can see th Spherical body is directly proportional to densities ofthe body andthe fui greater than that of body (ie, o >p) fhismeans thatthe body instead of Bir bubbles rise up in water. at emia veloc ofa ihe dference nn ol the dest of fis ne terminal veces eat fling moves ward These fromple 13.26 With what terminal velocity will an air bubble 08 mm in diameter rise in a liquid of viscosity 0.18 Nan? and specific gravity 0.9. Density of air is 1.293 kgm” So The terminal velocity of the bubble is given by, Pip-olg 7) 04x 107 m, vp Check point 33) Units of cGetticient of viscosity are (@) Nms=* (0) Nm’s* (9 Nm (@) None of these 2 As the temperature of water increases, is viscosity (@)remains unchanged decreases (increases (increases or decreases depending on the external Pressure 3. The rate of flow of quid in a tube of radius r, length /, whose nds are maintained at a pressure difference pis V = here nis coeticient ofthe wscosity and Wis @a 4 o% OF 4 Two capilay tubes of the same length but diferent rac 4 89d fare fitted in paraliel to the bottom of vessel. The Bressure head isp. What should be the radius ofa single ©) 16 mechanics, (05 Chapter 13 : Flt 6 =0.9x 10° kgm™ p =0.15,Nsm~ and g =9.8 ms~ Substituting the values, we have, ous 2933 gen Note Here negative sign implies that the bubble will rise up. Example 13.27 Two spherical raindrops of equal size are falling vertically through air with a terminal velocity of 1 ms What would be the terminal speed, if these two drops were t0 coalesce to form a large spherical drop. Sok As, vpecr? oli) Let rbe the radius of small raindrops and R the radius of large drop. Equating the volumes, we have dn o(ee) R=OP ro 2 * -(8) =? vp UF vp =P vp = 2)! 0.0) ms tube that can replace the two tubes so that the rate of flow is same as before @ itn @ 42 han Ase (2) None of hese 5. The terminal velocity v of a small stee! ball of radius r falling under gravity through a column of viscous liquid of Coetticient of viscosity n depends on mass of the ball m, acceleration due to gravity g, coefficient af viscosity 1 and radius r. Which of the following relations is dimensionally correct? @ vee (b) vemg nr Ova (@) ve DOD wr 7 6. The ratio of the terminal velocities of two drops of radii R and Ri2is (@) 2 © 4 (a sn) OBJECTIVE PHYSICS Vol. 7 7. An air bubble rises trom the bottom ot a lake of large depth. The rising speed of air bubble will (@) go on increasing til it reaches surface (©) go on decreasing tl it reaches surface (©) increases in two beginning, then will become constant (@) be constant all throughout 8. Two equal drops of water are falling through air with a steady velocity v. If the drops coalesce, the new velocity will be (@) av (o) vv (2% Oe 9. From amongst the following curves, which one shows the variation of the velocity v with time ¢ for a small sized spherical body falling vertically in a long column of a viscous liquid 6) Surface tension Surface tension is the property of liquid at rest by virtue of which a liquid surface tends to occupy a minimum surface area and behaves like stretched membrane. ‘A small drop of liquid is spherical in shape, because for given volume, sphere has minimum surface area. ‘This stretched behaviour is due to net downward force acting on a molecule on the surface of liquid as shown in figure. A molecule well inside the liquid experiences no net force. Fig. 13.31 Forces between molecules of a liquid Now consider a line AB on the free surface of the liquid. The small elements of the surface on this line are in equilibrium because they are acted upon by equal and opposite forces, acting perpendicular to the line from cither side as shown in figure. Fig, 13.32 Force on a line at the surface of a liquid 8 » ie ae , © @ 410. A spherical ball of radius 3.0 x10" m and density 10 kgm fale froely uner gravity through a distance h before enters a tank of water, If after entering the water the Velocity or te bail does not change, what isthe value of (Viscosty of water is 9.8%10°° Nsm~). (@) 1.6510? m (©) a65x10" m (b) 2.65%10? m (@) 1.45%102 m The force acting on this line is proportional to the length of this line. If /is the length of imaginary line and F the total force on either side of the line, then or From this expression, surface tension can be defined as the force acting per unit length of an imaginary line drawn on the liquid surface, the direction of force being perpendicular to this line and tangential to the liquid surface. It is denoted by S and it is a scalar quantity. Note The surface tension ofa particular liquid usualy decreases aS temperature increases. To wash clothing throughly, water must © forced through the tiny spaces between the fibres. This requires increasing the surface area ofthe water, which iscficlt 9 62 because of surface tension. Nence, hat water and soapy water S better for washing and dimension of surface tension SI units of surface tension = Nm~ CGS unit of surface tension = dyne cm™ Dimension of surface tension Force _[MLT~*] Length wu =[ML°T-?) 1328 A liquid is kept in a beaker of radius 4 e fs ett of he bev onthe face af he ater ama jorce bv which the surface on one side of the di F face on the other side. ae Mie the surly Surface tension of liqui me sNin" Of tiquid ‘re length of the diameter is wo =8em=0.08m tension is S=F / Thus, (0.075Nm™")x (0.08) = 6x 10° N ‘The surface Fe surface energy spe free surface of a liquid always has a tendeney to Marract and possess minimum surface area. To increase gfe surface area of the liquid work has to be done. This ire done is stored in the surface film of the liquid as its ferential energy. This potential energy per unit area of the Pface film is called the surface energy. Hence, the surface energy may be defined as the imount of work done in increasing the area of the urface film through unity. Thus, ‘Work done in increasing the surface area ee Increase in surface area The SI unit of surface energy is Jm~ Relation between surface energy and surface tension Consider a rectangular frame PQRS. Here, wire QR is movable, A soap film is formed on the frame. The film hulls the movable wire QR inward due to surface tension. Force _F = pr =5x2 Length 2! As, surface tension = wae w ullibsium position of the film Stretched positon ofthe fm Fig. 13.33 UF QR is moved through a distance z by an external force F ‘ety slowly, then some work bas to be done against his force. “. External work done = Force x Distance = Sx xz Increase in surface area of film = 2!* = [As soap “Work done. surface area Surface area _ Pe) ilm has two sides] Chapter 13 : Fluid mechanics (on So, value of surface energy of liquid is numerically equal to the value of surface tension. Note 1. Work done in forminga dropW = S x 4x1? (as aA 2. Work done in forming a bubbleW = 5 x 4nr? x 2(As bubble in air has two surfaces) = nr? = 0= 4007) Example 13.29 How much work will be done in increasing the diameter of a soap bubble from? cm to 5 cm. Surface tension of soap solution is 30% 10~! Nin“. Sol. Soap bubble has two surfaces. Hence, WeTAA Here, AA =2[An (2.5107)? - (01077773) = 1.32% 107 m? Work done W'= (3,0%10"%)(132%10) J=3.96%10~ J Example 13.30 Calculate the energy released when 1000 small water drops each of radius10~' m coalesce mme large ‘drop. The surface tension of water is7.0 x 10 Nm”. Sol, Let rbe the radius of smaller drops and R of bigger one. ‘Equating the initial and final volumes, we have 4 gr? = 10000(4 me? rind «xo001($ ) or R=10r=(10\10-7)m or R=10%m Further, the water drops have only one free surface. ‘Therefore, AA= 4a? - 1000}(4xr?) = 4x (10°? -00°)0"7F] -36n110-)m? Here, negative sign implies that surface area is decreasing. Hence, energy released in the process. U=TIAAl = (71072986 nx 10°) J = 7.9% 10-7 J Excess pressure ‘Due to surface tension, a drop or bubble tends to contract and so compresses the matter enclosed. This in turn increases the internal pressure which prevents further contraction and equilibrium is achieved. So, in equilibrium the pressure inside a bubble or drop is greater than outside and the difference of pressure between two sides of the liquid surface is called excess pressure. 1. Excess pressure inside soap bubble ‘A soap bubble consists of two spherical surface films with fa thin layer of liquid between them. Because of surface tension, the film tend to contract in an attempt to minimize their surface area. But as the bubble contracts, it compresses s) OBJECTIVE PHYSICS Vol. 1 the inside air, eventually increasing the interior pressure to a level that prevents further contraction. Fig, 13.34 A soap bubble of radius R Excess pressure inside the soap bubble in terms of its radius R and the surface tension T of the liquid is given by mo Fig, 13.35 Pressures inside and outside of a ‘soap bubble ar or Ap = p—Po R Example 13.31 0.04 cm liquid column balances the excess pressure inside a soap bubble of radius 6 mm. Evaluate density of the liquid. Surface tension of soap solurtion = 0.03 Nm! Sol. The excess pressure inside a soap bubble is 4 0.03Nm" 6x 10m “The pressure due to 0.04 cm of the liquid column is Ap =hpg = (0.04 x 10"*m)p d0ms*) Example 13.32 Two separate soap bubbles (radii 0.004 m and 0.002 m) formed of the same liquid (surface tension 0.07 Nm) come together to form a double bubble. Find the radius and the sense of curvature of the internal film surface common to both the bubbles. Sol, Excess pressure inside first soap bubble A= Pot a Excess pressure inside second soap bubble aT P2=Po+—— a % ” nish mA i.e, pressure inside the smaller bubble will be more. The ences pressure =p-anaT (i= rencn-af) ‘This excess pressure ats from concave to conver side, the interface willbe concave towards smaller bubble and conte towards larger bubble, Let Rbe the radius of interface thee ar 7 fi) From Eqs. (i) and (i), ff, (0.004.002) =n (0.004 = 0.002) 2. Excess pressure inside a liquid drop A liquid drop has only one surface film excess pressure inside the liquid drop is given by or Ap R Some Important points regarding excess pressure (i) If we have an air bubble inside a liquid, a single surface is formed. There is air on the concave side and liquid on the convex side. The pressure in the concave side (that is in the air) is greater than the pressure in the convex side (that is in the liquid) by an amount 27/R. Fig. 13.36 Excess pressure n an air bubble inside a liguid ar R i The above expression has been written by assuming Py to be constant from all sides of the bubble. For small size bubbles this can be assumed. PoP, a jy From the above discussion, we frement. The pressure on the conser caer spherical liquid surface is greater ti than the convex Bie by 27/R as shown below. Ro a = py BE Pa- pa A Fig. 13.37 (ii) Excess pressure inside a soap bubble can also be understood in terms of excess pressure inside a curved surface a shown below. Fig. 13.38 ane = pe =r 1 PZ m=z a maz (6) If two bubbles of different sizes are connected with each other through a thin tube (as shown in figure) then the air will rush from smaller to larger bubble (pressure inside smaller bubble will be greater than Pressure inside larger bubble), so that the smaller will shrink while the larger will expand till the smaller bubble reduces to a droplet. Tube 1 Excess pressure Ap = p, ‘Larger Smaller bate Dubble Fig. 13.39 Sample 13.33 What should be the pressure inside a small air $F 0-1 mm radius stated fst below the water surface? face tension of water = 12% 10-2 Nm°! and atmospheric "e=1.013%10° Nm? Surface tension of water T'=72% 10°? Nm 'S of air bubble R= 0.1 mm =10“ m — Chapter 13 : Fluid mechanics (618 ‘The excess pressure inside the air bubble is given by, ar Pom Peau ide the air buble, py = + 2 Substituting the values, we have 4 (2x 7.2% 107) 2 = (1.013 « 10° pom 6 y+ Gate 027% 10° Nm? Angle of contact ‘The surface of liquid near the plane of contact with another medium is in curved shape. ‘The angle of contact is defined as the angle that the tangent to the liquid surface at the point of contact makes with the solid surface. It is denoted by 0. The value of contact angle determines whether a liquid will spread on the surface of a solid or it will form droplets on it. When a liquid is put on a solid surface, then there are three interfacial tensions at all the three interfaces; liquid-air, solid-air and solid-liquid denoted by S,,, 5,, and Sy. At the line of contact, the surfaces between the three media must be in equilibrium. The following cases arises (i) If the surface tension at the solid-liquid Sy , interface is greater than the surface tension at the liquid-air Sj, interface, i.e., $y > Sy, then 0 > 90°(the angle of contact is obtuse angle). Sy Se Sr Fig. 13.40 The molecules of a liquid are attracted strongly to themselves and weakly to those of solid. It costs a lot of energy to create a liquid- solid surface. The liquid then does not wet the solid. ¢.g., Water-leaf or glass-mercury interface. (a) If the surface tension at the solid-liquid 5, is less than the surface tension at interface, ie., Sy < Si, contact is acute angle). S Fig. 13.41 interface the liquid-air S,, then 0< 90° (the angle of 620) OBJECTIV- PHYSICS Vol. 1 The molecules 0. of solid and weak energy to create a solid. .9., When soap or of contact becomes + + liquid are strongly attracted to those id-solid surface and liquid wets the gent is added to water, the angle Shape of liquid surface The curved surface of the liquid is called meniscus. The shape of the meniscus (convex or concave) is determined by the relative strengths of cohesive and adhesive forces. The force between the molecules of the same material is known as cohesive force and the force between the molecules of different kinds of material is called adhesive force. When the adhesive force between solid and liquid molecules is more than the cohesive force between liquid-liquid molecules (as with water and glass), shape of the meniscus is concave and the angle of contact @ is less than 90°. In this case the liquid wets or adheres to the solid surface. Anale of| Angle of 5 Fig. 13.42 When the adhesive force between solid and liquids molecules is less than the cohesive force between liquid-liquid molecules than shape of meniscus in convex. Note the angie of contact between water and clean glass is zero and that between mercury and clean glass is 137° Capillarity ‘The term capilla means hair which is Latin word. A tube of very fine (hair-like) bore is called a capillary tube. Mercury Fig, 13.43 Elevation and depression liquid in a capillary tube If a capillary tube of glass is dipped in liquid like water, the liquid rises in the tube, but when the capillary tube is dipped ina liquid like mercury, the level of liquid fails in the tube, This phenomenon of rise or fall of a liquid in the capillary is called eapillarity. Some examples are (i) Small capillaries in fibers of towels rocks wate, our skin Tom, (Gi) In trees sap rises in stem due to capillary action, Formula of capillary rise When a capillary tube is dipped in liquid then the jog of liquid in capillary tube rises or falls Wart. free suriyce of liquid outside the capillary. a” R bs Fig. 13.44 Shope of meniscus ‘This phenomena of rises or fall of liquid is called capillary action, The formula in capillary motion is 2F cos pg where h ight of liquid column rises or falls radius of capillary tube lensity of liquid .cceleration due to gravity © = angle of contact and T = surface tension The result has following notable features, (i) If the contact angle @ is greater than 90°, the term c0s@ is negative and hence, h is negative. The ‘expression, then gives the depression of the liquid in the tube. (i) Suppose a capillary tube is held vertically in a liquid which has a concave meniscus, then capillary rise is given by, 2 cos@_ wr m9 Rpg or hp = 20 °9 Note 1 the tubes ent es than adhesive force between oil and slase (b) cohesive force of oil > cohesive force of water (0) oil repels water (a) cohesive force of water > adhesive force between water and oil molecules 10. Which of the fact is not due to surface tension? (a) Dancing of a camphor piece over the surface of water (0) Small mercury drop itself becomes spherical (©) A liquid surface comes at rest after stirring {@) Mercury does not wet the glass vessel 11. On mixing the salt in water, the surface tension of water will (b) decrease (@) None of these 12, If two identical mercury drops are combined to form a single drop, then its temperature will (a) decrease () increase () remains the same (€) None of these (2) increase (0) remain unchanged 18. A water proofing agent changes the angle of contact {@) from acute to 90° (©) from obtuse to 90° (©) from an acute to obtuse value (d) from an obtuse to acute value 14. Along a streamline. (@) the velocity of a fluid a fluid particle remains constant (© the velocity of all fluid part ed co festonis Constant | amuses cosing a give (0) the valocity of i : 2 Yalocity ofall fluid particles ata given instant (@) the speed of a fluid particle remains constant ra 16. na 18, 19, 20, a streamline flow Inv the speed of a particle always remains same {p) the velocity of a particle always remains same 1 the kinetic energies ofall particles ariv point are the same (g) the potential energies of all the igven point are the same ing ata given Particles arriving at a {A viscous fluid is flowing through a cylindrical tube. ‘The velocity distribution of the fluid is best. represented by the diagram @ © Ul @ (a) None of these Which of the following diagrams does not represent a streamline flow? )))) A body measures 5N in air and 2N when put in water. The buoyant force is @7N ON ©@3N {@) None of these The reading of spring balance when a block is suspended from it in air, is 60 N. This reading is changed to 40 N when the block is immersed in water. The specific gravity of the block is @ 3 ) 2 06 32 A hole is made at the bottom of the tank filled with Water (density = 1000 kgm”). If the total pressure at the bottom of the tank is three atmospheres (1 atmosphere = 10° Nm“), then the velocity of efflux is nearest to © 900 ms" ©) 206 ms (9 J600 ms ® V500 ms — Chapter 13 : Fluid mechanics (ws 21. An iron block is on a boat which floats ina pond. ‘The block is thrown into the water. The level of water in the pond will be (a) equal to the earlier level () less than the earlier level (6) more than the earlier level {(@ depends on how large the block is 22. The correct curve between the height or depression h of liquid in a capillary tube and its radius ris | Ke @ 3) h h iC) | @ 23. When two capillary tubes of different diameters are dipped vertically, the rise of the liquid is {@) same in both the tubes (©) more in the tube of larger diameter {6) less in the tube of smaller diameter (@) more in the tube of smaller diameter 24. The liquid in the capillary tube will rise if the angle of contact is (@) 120") 90° (© obtuse (d) acute 25. The surface tension of liquid at its boiling point (a) becomes zero () becomes infinity (6) is equal to the value at room temperature (@) is half to the value at the room temperature 26. When the temperature is increased the angle of contact of a liquid? {@) increases () decreases (c) remains the same (4) first increases and then decreases 27. The surface tension of a soap solution is 2x10 Nm” work done is (a) 4xx 10°) (©) 12x10} To blow a bubble of radius 1 cm, the () Bx x10 (@) 16x 10%} 28, A liquid rises in a vertical tube. The relation between the weight of the liquid i tension ofthe liquid T and ratius of the tare oe given by, if the angle of contact is zero - @) Wen? r (OW =2r' aT ~ 624) OBJECTIVE PHYSICS Vol. 1 29, When two soap bubbles of radius 1, and r2{r, >") coalesce, the radius of curvature of common surface an 24 % © @ nts aoa 80. The excess pressure due to surface tension in a spherical liquid drop of radius r is directly proportional to @r 2 @rt @ r? 31. If two soap bubble of different radii are in communication with each other (a) air flows from larger bubble into the smaller one () the size ofthe bubbles remains the same (©) air flows from the smaller bubble into the large one and the larger bubble grows at the expense of the smaller one (@) None of the above ‘82. Assuming that the atmosphere has the same density anywhere as at sea level (p = 1.3 kgm™) and gto be constant (g = 10 ms”). What should be the approximate height of atmosphere? (Go =1.01x 10° Nm“) (a) 6 km (b) 8km (©) 12km (d@) 18 km 83. If the angle of contact is less than 90° the pressure just inside the surface of a meniscus (a) is less than atmospheric pressure (b) is greater than atmospheric pressure (€) is same as the atmospheric pressure (d) None of the above ‘84, The spiders and insects move and run on the surface of water without sinking because (a) elastic membrane is formed on water due to property of surface tension (B) spiders and insects are lighter (€) spiders and insects swim on water (d) spiders and insects experience upthrust 35. If two glass plates are quite nearer to each other in water, then there will be force of (a) attraction (b) repulsion (c) attraction or repulsion (4) Neither attraction nor repulsion 86. A tall cylinder is filled with viscous oil. A round pebble is dropped from the top with zero initial velocity. From the plot shown in figure, indicate the one that represents the velocity (v) of the pebble as a function of time (0). @- | oy @y| + ae 37. A tank if filled with water upto height H. When a hole is made at a distance h below the level of water ‘What will be the horizontal range of water jet? fa) 2Jh(H —h) (b) 4yA(H +h) (o 4h) (a) an +h) 38, A tank is filled to a height H. The range of water coming out of a hole which is a depth H/4 from the surface of water level is 2H BH @ oe o) cog ‘39. The density of ice is x g cm™ and that of water is y gcm™ when m gram of ice melts, then the change 3H iq) 3H OH at in volume is @ my-x ) 2 m (©) my(y-x) @ 2-2 yx 40. Two capillary tubes P and Q are dipped in water. The height of water level in capillary P is 2/3 to the height in Q capillary. The ratio of their diameters is @ 2:3 (0) 3:2 (3:4 @ 41. In a surface tension experiment with capillary tube Water rises upto 0.1 m. If the same experiment is repeated on an artificial satellite, while is revolving around the earth, water will rise in the capillary tub? upto a height of (@) 0.1m ©) 02m {c) 0.98 m (@) full length of the capillary tube 42, Two Parallel glass plates are dipped partly in the {avid of density d keeping them vertical. If the stance between the plates is x surface tension for 7 liquid is T and angle of contact 9, then rise of liquid between the plates due to capillary will be Tcos® 0 (a) Tes® gy 27 cose ar T cos xd = © os xdgcos8 do > 43, A balloow bas volume of 2000 ms filled with hydrogen ( = 0.009 gL), If the density of air is 129 ¢L '. it can lift a total weight of (a) 600 ky (0) 2400 kg (©) 300 kg (@) 1800 ig _ Araft of mass M = 600 kg floats in alm water ith J emsubmerged. When a man stands on the ral: g.4 cmare submerged, the man’s mass is : @) 30k) Gk (C) OOK) 120 ky . Acylinder of mass M and density d, hanging fvum a string, is lowered into a vessel of cross-sectional area ‘A containing a liquid of density a (dy < dy) winil it is fully immersed. The increase in pressure at the bottom of the vessel is oy Msg yy MP w Mb dA a ia Which of the following statements are true in case when two water drops coalesce and make a bigger drop? (a) energy is released (b) energy is absorbed {@) the surface area of the bigger drop is greater than the sum of the sueface areas of both the drops (@) the surface area of the bigger drop is same that of the sum of the surface areas of both the drops A piston of cross-sectional area 100 cm? is uscd in a hydraulic pressure to exert a force of 107 dyt the water. The cross-sectional area of the ot piston which support a truck of mass 2000 kg is (@) 9.8% 10? em? (0) 9.8% 10° em? (©) 1.96% 10° em? (4) 1.96% 10° em? An ideal fluid flows through two pipes of circular cross-section with diameters 2.5 em and 3.75 em connected one after another. The ratio of the velocities in the two pipes is () 9:4 () 3:2 (VB i V2 (@ V2:N8 The angle of contact at the interface of water-glass is 0, ethyl alchol-glass is 0°, mercury-glass is 140° and methyliodide-glass is 30°. A glass capillary is put in a trough containing one of these four liquids. Tt is observed that the meniscus is convex. The liquid in the trough is (@) water () ethylalcobol (©) mercury (@) methyliodide A uniformly tapering vessel is filled with a liquid of density 90 kgmr®. The force that acts on the base of the vessel due to the liquid (excluding atmospheric force) is (g = 10 ms~*) — Chapter 13 : Fluid mechanics — Aven = 10° m? (0) 3.6N () 72N (© 9.0N (a) 12.6 51. For the arrangement shown in the figure, the force at the bottom of the vessel is 100 em (2) 200N @) 100N@) 20N@) 2N 52. An object of uniform density is allowed to float in water kept in a beaker. The object has triangular cross-section as shown in the figure. If the water pressure measured at the three points A, B and C below the object are ps Pp and pc respectively. ‘Then (0) pa> Pa? Pe © Pam Pa= Pe ©) pa> Pa % © 4 Ded © Ore @ 68. An object weights m, in a liquid of density d, and that in liquid of density d, is m,. The density of the object is (ay Mata = md, ® ™—m my =m, (Madam (o Mier a m— my 69. AAn ice-cube of density 900 kgm” is floating in Water of density 1000 kgm”, The percentage of volume of ice-cube outside the water is @) 208) 808 108 (@) 90% A es nthe figure shown, 10. p. 2» wore WP>% © P=P (@) p=0 si. balloon has volume of 1000 m’. It is filled with hydrogen. (P = 0.09gL"'). If the density of air is 129 gL, it can lift a total weight of (b) 1200 kg (@) 1800 kg. sa, A boat having a length 3 m and breadth 2 m is floating on a lake. The boat sinks by 1 cm when a fon gets on it. The mass of the man is ekg WO T2kg (9) S2ke — (a) GS ky 78. Asmall block of wood of relative density 0.5 is Abmerged in water. When the block is released, it ats moving upwards, the acceleration ofthe block is (= 10 ms“) (@) Sms? (b) 10ms? 1H. A raft of wood of mass 120 kg floats in water. The ‘weight that can be put on the raft to make it just sink,should be (dja, = 600 kgm”) @) 80kg 6) 50kg (0) GOke—) 30 kg 18, An object of w and density p is submerged in liquid of density o, its apparent weight will be @ 6-9) 0) @-o)/w mie} we 6. A stone of relative density K is released from rest on the surface of a lake. If viscous effects are ignored, the stone sinks in water with an acceleration of () ot -K) () gl +X) 1 1 of1-z) waltz) T. A body floats in a liquid contained ina beaker. The whole system shown in figure is falling under gravity, the upthrust on the body due to the liquid is {a) zero. 8 ‘equal to weight of the body baat equal to weight of liquid disp! @) equal to earns ‘athe immersed part of the body 78 The relative density of ice s O9and that of #8 ‘water is 1.125, What fraction of the whole plumes of an iceberg appears above the surface of the sea’ f@ v5 b) 2/5 © a5 (o) 4/5 (2) T5ms* (@) 15s? 179, A metallic sphere floats in immiscible mixture of water (density 10° kgm”) and a liquid (density 8 10° kkgm™) such that its (2/3)pare is in Water and (1/3) part in the liquid. The density of the metal is * 20822 gm (©) 5000 kgm* (2) 2000 kgm 80. In the given figure, the velocity v3 will be er Ay = 022 me = a NK S y= Od m? (@) 2s () 4 ms (o1ms* (@) 3mst 81. A water tank standing on the floor has two small holes punched in the vertical wall one above the other. The holes are 2.4 cm and 7.6 cmabove the floor, If the jest of water from the holes hit the floor at the same point, then the height of water in the tank is (a) 10 (b) Sem (€) 20em (@) 48cm 82. A body of volume V and density p is initially submerged in a non-viscous liquid of density ¢ © p)- If itis rises by itself through a height hin the liquid. Its kinetic energy will (@) increase by hV (6—p)g_ (b) increase by hV + og Veg AVog (©) increase by “VP? (@) decrease by 83. The pressure of the gas in a cylindrical chamber is pg. The vertical force exerted by the gas on its hemispherical end is (@) po? (b) pone? (© 2ponr? (@) porr® 84, Water rises in a capillary tube to a certain height such that the upward force due to surface tension is balanced by 75x 10~ N force due to the weight of the liquid. If the surface tension of water is 6x10 Nm“, the inner circumference of the capillary must be (@) 1.25107? m (©) 65x10 m () 0.5010 m (@) 12.5107? m 28) OBJECTIVE PHYSICS Vol. 1 7 85. Water rises upto 10 cm height in a long capillary tube. If this tube is immersed in water so that the height above the water surface is only 8 em, then {a) water flows out continuously from the upper end {b) water rises upto upper end and forms a spherical surface (©) water only rises upto 6 cm height (@ water does not rise at all 86. If a capillary tube is dipped into liquid and the levels of the liquid inside and outside are same, then the angle of contact is (a) 120° (b) 90° (©) 45° (a) 30° 87. Water rises to a height of 30 mmin a capillary tube. If the radius of the capillary tube is made 3/4 of its previous value. The height to which the water will tise in the tube is @ 30mm 0) 20mm (©) 40mm) 10 mm 88. A vessel, whose bottom has round holes with diameter of 1.0 mmis filled with water. The maximum height to which the water can be filled without leakage is (Surface tension of water = 75 dyne cm“, g = 1000 cms) fa) 100 cm (b) 75cm (c) 50cm (d) 30cm 89. A long cylindrical glass vessel has a small hole of radius r at its bottom. The depth to which the vessel ‘can be lowered vertically in the deep water bath Gurface tension T ) without any water entering inside is (a) 4T/prg (b) 3T/prg (©) 27 /prg (@) T/prg 90. If a water drop is kept between two glass plates, then its shape is o | ) o 0 ) ( (@) None of these 91. Two small drops of mercury each of radius r form a single large drop. The ratio of surface energy before and after this change is 2:27 )2P.1 2st (asd 92. A non viscous liquid is flowing through a frictionless duct, cross-section varying as shown in figure. Which of the following graph represents the variation of pressure p along the axis of tube p : ° wf of x x i ° © @ N\A x x 93. There is hole of area a at the bottom of a cylindsica of area A. Water is filled upto a height h and water flows out in second. If water is filled to a height 4h it wil flow out in time t t at (b) 2¢ (e) at @; 94, A container has a small hole at its bottom. Area of cross-section of the hole is A, and that of the container is A,-Liquid is poured in the container at a constant rate Qm’s~!. The maximum level of liquid in the container will be @) e 2 ¢ om “ae “nan @aee 96. Air stream flows horizontally past an aeroplane wing of surface area 4 m?. The speed of air over the top surface is 60 ms"! and under the bottom surface is 40 ms” The force of lift on the wing is (density of air = 1 kgm™) (@) 800N (b) 1000N (c) 4000N — (d) 3200 N 96. Water from a tap emerges vertically down with an initial speed of 1.0 ms~!. The cross-sectional area of tap is10™ m?. Assume that the pressure is constant throughout the stream of water, and that the flow is steady. The cross-sectional area of the stream 0.15 ™ below the tap is, (a) 5.0x 10 m? (0) 1.0x 107° m? (0 5.0% 10° m? (@) 20% 107° m? 97, There are two holes 0, and O, in a tank of height #- The water emerging from Q, and OQ, strikes the Sround at the same points, as shown in figure. The! 0 @ Hah +h, (0 H= Jig, @) H=h,-h, (@ None of these P ee -qwo rain drops of same radius r falling with terminal wMocity ¥ merge and from a bigger drop of radius R. ‘The terminal velocity of the bigger drop is ook R wee fev (@2v ‘Acubical vessel of height 1m is full of water. What is the amount of work done in pumping water out of. the vessel? (Take, 9 = 10ms~*) (a) 1250) () 5000 J (0) 1000} (@) 2500] 100, A cubical block of steel of each side equal to 1 is floating on mercury in vessel. The densities of steel and mercury are p, and p,,. The height of the block above the mercury level is given by veep ee) wb) bt 101. A cylindrical tank contains water up to a height H. If the tank is accelerated upwards with acceleration a, the pressure at the point A is p,. If the tank is accelerated downwards with acceleration a the pressure at A is p).Then (9 p, (d) Data insufficient 102. A metal sphere connected by a string is dipped in a liquid of density p as shown in figure. The pressure at the bottom of the vessel will be, (py = atmospheric Pressure) () P=P + poh ©) p> Po + Pah tog, «P p) to a depth h and then released. The height above the surface of water up to which the ball will jump is ) (gn js ° fw 2% a 5 o(1-24)a ° ‘A U-tube of base length 1 filled with same volume of two liquids of densities p and 2p is moving with an acceleration a on the horizontal plane. If the height difference between the two surfaces (open to atmosphere) becomes zero, then height h is given by @ a @ 2al © 36 @ oy ) OBJECTIVE PHYSICS Vol. 1 109.In a U-tube experiment, a column AB of water is balanced by a column CD of paraffin. The relative density of paraffin is hah @ 110. A U-tube of uniform cross-section shown in figure is partially filled with liquid I. Another liquid It which | does not mix with I is poured into one side. The liquid levels of the two sides is found the same, while the level of liquid I has risen by 2 cm. If the specific gravity of liquid I is 1.1, then specific gravity of liquid 1 must be @12 wrt 1.3 @1.0 111. A closed rectangular tank is completely filled with water and is accelerated horizontally with an acceleration towards right. Pressure is la 2] la ¢| () maximum at, and (ii) minimum at @) (@BGwD ©) @C wD (© wa Gc @ OB GA 112. The liquid inside the container has density p. Choose the correct option. (@) Pa- Pe (©) Pe — Py = V2pgL © Pe-P (@ Pa Po =OpaL 118. A tank contains water on top of mercury as shown in figure. A cubical block of side 10 cmis in equilibrium inside the tank. The depth of the block inside mercury is (RD of the material of block = 8.56, RD of mercury = 13.6 f@6em Sem =O 7em A) Bem 414. 1f Tis the surface tension of a liquid, the energy needed to break a liquid drop of radius R into 64 drops is, 7 (@) 6xkT (omer (Q)12nR*T (@ 6nR°T (eo) 4n'T 115. Two solid spheres of same metal but of mass M ang 8M fall simultaneously on a viscous liquid and their terminal velocities are v and nv, then value of n is (a) 16 (b) 8 (4 @2 116. A cubical block is floating in a liquid with half of its volume immersed in the liquid. When the whole system accelerates upwards with acceleration of 49/3; the fraction of volume immersed in the liquid will be (a) 1/2 (b) 3/8 (c) 2/3, (d) 3/4 117. A cylindrical vessel is filled with a liquid up eg toa height H. A small hole is made in the vessel at a distance y | below the liquid surface | as shown in figure. The liquid emerging from the hole strike the ground at distance x (@) xis equal if hole is at depth y or H — y (b) xis maximum for y =H/2 (©) Both (a) and (b) are correct (@) Both (d) and (4) are ‘wrong, 118. Water flows along a horizontal pipe whose cross-section in not constant. The pressure is 1 cm of Hg where the velocity is 35 cms. At a point where the velocity ® 65 cms”, the pressure will be (2) 0.89 cm of Hg (b) 8.9 cm of Hg (00.5 cm of Hy (@) Lem of Hg ae of lengths L and 20 and of radius and 2R are connected in seri net rate of flow of fluid through thea will b Grae! of the flow through single capillary, X = mpR* /8n2) 8 @ ox 2 7 9 ox ix 119, 5 opx “a a soap bubble is blown with the help of a ‘pechanical pump at the mouth of a tube. The pump woduces a constant increase per minute in the radius Prthe bubble, irrespective of its internal pressure. ‘The graph between the excess pressure inside the goap bubble and time ¢ will be ’ P 1204 o @ 121. Two soap bubbles of radii r, and rp equal to 4 em and 5 cm are touching each other over a common surface S,S (shown in figure). Its radius will be fa 4em — @) 20cm) Sem_—@) 4. em 122, A uniform long tube is bent into a circle of radius R and it lies in vertical plane. Two liquids of same volume but densities p and 6 fill half the tube. The angle 0 is 3 (stant ( ) oar(' ps8, © un! (:) ta (e 7 128. Two substances of relative densities Pi ‘mized in equal volume and the relative mixture is 4. When they are mixed inequal masses, the relative density of the mixture is 3. The values of e) y= Sandpa=5 (a) None of these 124, 125. 126. 127. 128. 129, 180. em thick with a vertical hole iilled through it is floating in a lake. The minimure frie out Hague to sop outa bucket Fal of water through the hole is (density of ice =09gem™) @ 05m — 10m 12m ‘A body of density p is dropped from rest from a height h into a lake of density o(6 > p). The maximum depth the body sinks inside the liquid is (neglect viscous effect of liquid) A large block of ice 10 @ 18m ee eee o-p o-6 o Acandle of diameter d is floating on a liquid in a cylindrical container of diameter D@ > > d) as shown. in figure. If it is burning at the rate of 2. cmh”, then the top of the candle will {@) remain atthe same height (0) fall at the rate 1 emh™ (c) fall atthe rate of 2 emh"* (@) go-up atthe rate of 1 emh™ 'A spherical object of mass 1kg and radius 1m is falling vertically downward inside a viscous liquid in a gravity free space. At a certain instant the velocity of the sphere is 2ms”!. If the coefficient of 1 viscosity of the liquid I units, then velocity of én ball will become 0.5 ms“ after a time. @In4s @&)2inds (J 3inds (d) Sin 2s ‘A wooden block of mass 8 kg is tied to a string attached to the bottom of the tank. In the equilibrium the block is completely immersed in water. If relative density of wood is 0.8 and g = 10 ms~, the tension T, in the string is @ 120N' @) 100N (©) 80N —@) 20N A metal ball immersed in alcohol weighs w, at 0°C and wy at 59°C. The coefficient of cubical expansion of the metal is less than that of alcohol. ‘Assuming that the density of the metal is large ‘compared to that of alcohol, it can be shown that @ w>w (b) wy =w, © 76 em. (©) 76 em (@) <76.cm 632) OBJECTIVE PHYSICS Vol. 1 181. The surface energy of a liquid drop is S. It is sprayed into 1000 equal droplets. Then its surface energy becomes @ s © 10S 1008 —_() 10008 182, An open tank containing non-viscous liquid to a height of 5 m is placed over the ground. A heavy spherical ball falls from height 40 m over the ground in the tank. Ignoring air resistance find the height to which ball will go back. Collision between ball and bottom of tank is perfectly clastic @) 45m) 35m) 40m_—(d) 200m 183, A large open tank has two holes in the wall. One is a square hole of side L at a depth y from the top and the other is a circular hole of radius R at a depth 4y from the top. When the tank is completely filled with water, the quantities of water flowing out per second from holes are the same. Then R is equal to @L2n WA W!L @ Ln 184. A piece of steel has a weight w in air, w, when completely immersed in water and w, when completely immersed in an unknown liquid. The relative density (specific gravity) of liquid is @ 2 wt ga @ A=“ wow, wou wow wow, 135. Two cylinders of same cross-section and length L but made of two materials of densities d, and d, are connected together to form a cylinder of length 2L. The combination floats in a liquid of density d with a length L/2 above the surface of the liquid. If d < dp, then 3 a a a waq wWf>4 Warde 186. A block of wood is floating on the surface of water in a beaker. The beaker is covered with a bell jar and the air is evacuated. What will happen to the block? (a) Sink alittle (b) Rise alittle (©) Remain unchanged (@) Sink completely 187.A beaker containing water is kept ‘on a spring scale. The mass of water and beaker is 5 kg. A block of mass 2 kg and specific gravity 10 is suspended by means of thread from a spring balance as shown. The readings of scales S, and S, are respectively Take, g =10ms* (@) 52 N and 20N 5 (b) 50 Nand 18N (©) 52.N and 18N (@) 52 N and 221N 138, A sphere of solid material of specific gravity g concentric spherical cavity and just sinks in ‘Then the ratio of the radius of the cavity to radius of the sphere must be Water, the outer Ne we > Aa « = oF 189. A cubical block of side 10 cm floats at the interfa. an oil and water. The pressure above that of atmosphere at the lower face of the block is Ce of 06 gee = ten|| ee tem (a) 200 Nev? (b) 680 N m* {) 400 Nm? (@) 800 Nm? 140. A liquid stands at the plane level in U-tube when at rest. If areas of cross-section of both the limbs are ‘equal, what will be the difference in heights h of the liquid in the two limbs of U-tube, when the system is, given an acceleration a in horizontal direction towards right as shown? | eae —— Le La Ba wt. mH @h2 @w : aH 9 Hg 141. A capillary tube is dipped in a liquid. Let pressure at points A,B and C be p 4, pgs pe respectively, then ©) pa= Ps Po the volume of an air bubble becomes 4b Mocs from the bottom of a lake to its: fits ae [umospheric pressure to be 75 cm of Hg and density of water to be 1/10 of the placer = mercury, the depth of the lake is ou, a. 48. Figure shows how the stream of Fier emerging rom a faucet necks donw as it falls. The area changes from Ag to A through a fall of h. At what rate does the water flow from the tap? 2ghA? ha? (a) A032 a? () i 4-4 Moves a oat oa] ae 2 eos 144.A ball of relative density 0.8 falls into water from a height of 2 m. The depth to which the ball will sink is (neglect viscous forces) fa) 8m (b) 2m () 6m @) 16m 145. A pump is designed as a horizontal cylinder with a piston area A and an outlet orifice arranged near the axis of the cylinder. Find the velocity of outflow of liquid from pump, if the piston moves with a constant velocity under the action of a constant force F. The density of liquid is p La [ar [Ap Ap. o~ of of of E «°F ef 2 146.1f cross- sectional area of limb I is A, and that of limb lis Ag then velocity of the liquid in the tube will be, (cross- sectional area of tube is very small) ) PBoe— 0 2 eH = 9 AT (@) None of these 148.A thread is tied slightly loose to Chapter 13 : Fluid mechanics 633 147. Two capillaries of same length and radii in the ratio 1:2 are connected in series. A liquid flows through them in streamlined condition. If the pressure across the two extreme ends of the combination is Lm of water, then pressure difference across first capillary is (a) 9.4m (0) 49m (© 049m (@) 0.94. m a wire frame as in figure and the frame is dipped into a soap solution and taken out. The frame is completely covered with the film. When A is pricked {@) thread will become concave on seeing from side A () thread will become concave on seeing from side B (©) thread will become straight (@) theead will remain as itis A tank filled with water has two taps to exhaust and pour. A hollow spherical ball is half submerged in water. Through one tap, water is taken out and through another tap, a liquid of density double the density of water is poured in tank such that volume of liquid in tank remains constant. Sphere will (a) go down () go.up. (6) maintain same height (d) sink to bottom . A container has two immiscible liquids of densities py; and p2 p,). A capillary tube of radius r is inserted in the liquid so that its bottom reaches upto, the denser liquid. The denser liquid rises in the capillary and attains a height h from the interface of the liquids, which is equal to the column length of the lighter liquid. Assuming angle of contact tobe zero, the surface tension of heavier liquid is (a) 2rpzgh © 52 -piok (@) 2x 2 ~p,)gh ~ G34) OBJECTIVE PHYSICS Vol. 1 151. A ball of mass 1 kg falls from a height of Sm_ above the free surface of water. The relative density of the solid ball is s = 2/3. The ball travels a distance of 2m under water and becomes stationary. The work done by the resistive forces of water is @) -50) ) -20} © 40) @ -30) 152, Under isothermal condition two soap bubbles of radii 1 and rz coalesce to form a single bubble of radius r. The external pressure is po. The surface tension of the soap in terms of the given parameters is 3 (yy Pal =a - 02) (2B =) 4qeed—r) ai +r) womereed) aged er) 153. A bent tube is lowered into the stream as shown in the figure below. The velocity of the stream relative to the tube equal to v. The closed upper end of the tube located at the hight hg. To what height will the water jet spurt? (a) None of these ortho P ‘0 art 154, A solid ball of density half that of water falls freely under gravity from a height of 19.6 m and then enters water. Upto what depth will the ball go. How ‘much time will it take to come again to the water: surface. Neglect air resistance and viscosity effects in water. (g = 9.8 ms~*) f@4s 6s Bs (@2s illed with water (density Alarge tank is filled with wa y 185: AON gga), A-small hole is made at a depen i below water surface. The range of water smi i yund. What extra of the hole is R on ground. Pressure be applied on the water surface so that the range become 2K (take 1 atm = 10° Pa and g= 10 ms-2 (@iatm =) 2atm = ()4atm —@) Satm 156. A wooden plank of length 1m and uniform, cross-section is hinged at one end to the bottom of a tank as shown. The tank is filled with water upto 4 height of 0.5 m. The specific gravity of the plank is 0.5. The angle 0 made by the plank in equilibrium position is fa) 30° (b) 45° (©) 60° (d) 90° 157. Air is blown through a pipe AB at a rate of 15 Lmin.~ The cross-sectional area of the broad portion of the pipe AB is 2 cm? and that of the narrow portion is 0.5 cm”. The difference in water level h is (density of air = 1.32 kg m™) (2)16 mm, (6) 10 mm, (0) 15 mm, (@) 32 mm » Medical entrance special format questions assertion and reason rection (Q. Nos. 1-20) These quest pirecti _ These questions consists of wo De Oi ae ens oni of Seuvring these questions you are required to choose any sof the following five responses. {a) Ifboth Assertion and Reason are correct and Reason is the correet explanation of Assertion, 4 Re (b) If both Assertion and Reason are true but Reason is nol the correct explanation of Assertion, 7 (© If Assertion is true but Reason is false, (d) If Assertion is false but Reason is true (e) If both Assertion and Reason are false. 1. Assertion Density of an incompressible liquid is constant. Reason An ideal fluid is incompressible. 2, Assertion In a freely falling liquid container, upthrust force is zero. Reason In freely falling case value of effective value of gis zero. 3. Assertion Bulk modulus of an incompressible liquid is infinite, Reason Compressibility is inverse of bull modulus. 4. Assertion If angle of contact is 0°, then liquid will neither rise nor fall in a capillary. Reason When angle of contact is 0°, surface is neither ‘convex nor concave inside the capillary. It is flat 5. Assertion Deep inside a liquid density is more than the density on surface. Reason Density of liquid increases with increase in depth, 6. Assertion Small water drops are spherical while bigger water drops are not. Reason In small water drops surface tension forces dominate while in bigger water drops gravity forces dominate. 7. Assertion When an ideal fluid flows through a pipe of non-uniform cross-section, then pressure Is more at that section where area is more if the pipe is horizontal. = Reason According to Bernoulli's theorem speed at broader cross-section will be less. 8. Assertion If ice is floating in water and it melts, then level of water remains unchanged. Reason When the ice is floating, weight of liquid displaced is equal to the weight of ice. = 9. Assertion An ice ball is floating in water. Some stone pieces are embedded inside the ice. When ice will melt, level of water will fall. Reason In floating condition, stone pieces will displace more liquid compared to the condition when they sink. 10. Assertion A solid is floating in a liquid of density 1p}. When the solid melts its density becomes p2 in liquid state. If p, > p.2 level of liquids will increase after melting. Reason In liquid state volume always increases after a solid melts. 11. Assertion A solid sphere and a hollow sphere both of same material are immersed in a liquid, then change in weight in both the spheres will be same. Reason Upthrust depends upon the volume of the solid immersed not the mass. 12. Assertion If water is filed in a balloon and this is immersed in water itself. Then volume of water displaced is equal to the volume of water filled in the balloon Reason Volume of a liquid displaced is equal tothe volume of solid immersed in that liquid. 18, Assertion At same level of same Liquid. pressure is always same, Reason When any fluid travels from a region of higher pressure to lower pressure {at same levels) it gains same speed 14. Assertion A solid is floating in a liquid. If temperature is increased and expansion of solid is ignored, then fraction of volume immersed will Reason By increasing the temperature density o liquid will decrease. 8 = a 15, Assertion A ball is released from the bottom of a tank filled with aliquid. moves upweands, Ie moving upwards upthrust will decree, Reason Density of ball is : auagh Density of ball is less than the density of 836) OBJECTIVE PHYSICS Vol. 1 Column Column) 16. Assertion A wooden plank is floating in two =e lpleee cere liquids as shown, Net force applied by liquid-1 on (Wane i vin () plank is zero. : - tee Reason Contribution in upthrust due to liquid -lis ‘ Speedef gad _9 more V;p1g. Where V, = volume immersed in liquid-land {@ Presare of nid © _Issame is the is the density of liquid -L : : 2, Two soap bubbles coalesce to form a single large drop. Match the following columns. Column I Column it (A) Surface energy in the process will (p) _ Tnerease (B)_Temperature ofthe deep will Decrease {B)_Temperature ofthe drop will __{@)_ Deer (@) Pressure inside the soap bubble will (©) Remain same aaarererrrs rr 3. There are two points A and Binside a liquid as shown in figure. Now the vessel starts moving ‘upwards with an acceleration a. Match the following columns. ain Reason Pressure at 1 is less than the atmospheric pressure. 18. Assertion A solid object of iron is dipped in water, both are at same temperature of 2°C. If the Column 1 Column IL temperature of water is increased by 2°C, then the a Pe a buoyancy force action of the object will increase. Bese Bel _ - Reason If we increase the temperature of water 9 Presur diference between (9) Decreate from 2°C to 4°C, then density of water will increase. Ignore expansion of solid sphere. (CQ Upthrust on an object inside () Remain same : the vessel wi 19. Assertion Mass of solid floating in liquid is m, 7 iid is my. Base area is A. Then (om +ma)9 and mass of 4. A tube is inverted in a mercury vessel as shown in pressure at bottom is py + figure. If pressure p is increased, then Reason Upward force on liquid from base of vessel WS is pA, where p is pressure at bottom. Columa Tl (eign e Match the columns ain 1. A liquid is flowing through a pipe of non-uniform EO Wildecrease cross-section. At a point where area of crose-section (© Pressure attem above O () Will main same of the pipe is less, match the following columns. - Chapter 13: Fluid mechanics (637 6. A cube is floating in liquid as shown in figure. Mateh the following columns. Column 1 Column I (A) If density of liquid decreases x will (@) Increase (B) size of cube is increased xwill (q) Decrease g.in the figure shown velocity of gud which comes In i etime of liquid to fall to ground ier a om con ground is R. Ifthe vessel taken tan raneyain, atch the following. (consider all eases ech might possible) Gotu Column 1 oo ip) Will increase tg) Wi decrease o R (o) Will emain sme (©) Ithe whole system is accelerated upwards x will (Remain same () Medical entrances’ gallery “ (Collection of questions asked in NEET & various medical entrance exams) 1, A rectangular film of liquid is extended from (dem x 2em) to (Sem x 4 x cm). If the work done is, 3x10}, the value of the surface tension of the liquid is INET 2016] (10.250 Nm (@) 0.125 Nm“ (0.2Nm" (@8.0Nm" 2. Three liquids of densities py. p2 and ps (with P, > Pz > py) having the same value of surface tension T, rise to the same height in three identical capillaries. The angles of contact ®,,,and 6, obey. INEET 2016] WE>Q>0;>0,20 050. <0, <0 <> 5 <0)<0, <0, Q>0:>0>> 3. Two non-mixing liquids of densities p and npin > 1) are put in a container. The height of each liquid is he A solid cylinder of length L and density dis put in this container. The cylinder Goats with im ems Vertical and length pL (p < 1) in the denser liquid. The density deqeal 7 INEET 2016] 12+ 0+ dplp (b) 2+ tm —Dplp (0+ —Dplp @arin+vele 4. A wind with speed 40 ms“! blows parallel to the roof of a house, The area of the roof is 250 m°. Assuming that the pressure inside the house is atmospheric bressure: the force exerted by the wind on the roof and the direction of the force will be (Pac = 1.2 kgm™) [CBSE AIPMT 2015) (a) 4.8 10° N, downwards (b) 4.8% 10° N, upwards (€)2.4x 10° N, upwards (@) 2.4% 10° N, downwards 5. The approximate depth of an ocean is 2700 m. The compressibility of water is 45.4x 107" Pa"! and density of water is 10°kgm™®. What fractional compression of water will be obtained at the bottom of the ocean? [CBSE AIPMT 2015} 0.8107 (W)1X107 —)1.2*107 1.4 x 107 Determine the height above the dashed line XX" attained by the water stream coming out through the hole is situated at point B in the diagram given below. Given that h=10 m, L = 2 manda =30%, TAIIMS 2015) 6. x x @1om = ®71m (Sm (3.2m 7. A water drop of radius 1 em is broken into 1000, equal droplets. What would be the gain in surface energy if the surface tension of water is 0.075 Nm"? (16.96 x 10"'J (04.24% 10°} (UK PMT 2015} (0) 8.48«10"} (212x105 638) OBJECTIVE PHYSICS Vol. 1 8. The lower end of a capillary tube is dipped into water and it is seen that the water rises through 7.5 em in capillary tube. What would be the radius of capillary of water is 7.5x 10" Nm !? ‘The contact angle between water and glass is O° and g=l0ms". [UK PMT 2015} @O2cm —W)O.tem (OA mm — (a) 0.2 mm A soap bubble of diameter a is produced using the soap solution of surface tension T. Find the energy required to double the radius of the bubble without change of temperature, [EAMCET 2015] @)2naT W)6naT W)3na’T (d)12na°T 10. A boat carrying a few number of big stones floats in a water tank. If the stones are unloaded into water, the water level [Kerala CEE 2015] (@) rises tll half the number of stones are unloaded and then begins to fall (©) remains unchanged (©) rises (@) fall tll half the number of stones are unloaded and then Degins to rise (6) falls 11. Choose the correct statement. (Kerala CEE 2015) (@) Terminal velocities of rain drops are proportional to square of their radii (©) Water proof agents decrease the angle of contact between water and fibres (6) Detergents increase the surface tension of water (@) Hydraulic machines work on the principle of Torriceli’s law (€) Venturimeter measures the flow speed of compressible fluids 12, A 20 cm long capillary tube is dipped vertically in ‘water and the liquid rises upto 10 em. If the entire system is kept is a freely falling platform, the length of the water column in the tube will be (WB JEE 2015] (a) Sem (b)10em — () 15cm (4) 20cm * 18. By sucking through a straw, a student can reduce the pressure in his lungs to 750 mm of Hg (density 13.6 gem) using the straw, he can drink water from a glass up toa maximum depth of [UP CPMT 2015) @10.2em (0) 75.3cm (€)13.6em (4) 1.96 em 14. If 1000 drops are combined to form a larger drop, then the ratio of surface energy of smaller drop to the larger drop will be (UP CPMT 2015) (@)1:100 (11000 (1:10 Ott 16, A solid floats such that its 1/3 part is above the water surface. Then, the density of solid is (UP CPMT 2015) @) 744 kgm? 1000 gn? tube, if surface te © = kgn"? (@) 910 kgm? 16. 11. 18, 19, 20. 21, 22, The amount of work done in blowing a soa P bub such that its diameter increases from d to D ig Me {3c surface tension of the solution) — iy, (a)2n Dd") S (b) x? -d?)s (nw? -d)5 (d) 8x? -a?)s With the increase in temperature, the angle of contact (a) increases [Manipal 201) (b) decreases (6) remains constant (@) first increases and then decreases, A bubble is at the bottom of the lake of depth h, As the bubble comes to sea level, its radius increases three times. If atmospheric pressure is equal to { metre of water column, then fis equal to (UK PMT 2014) 261) (251 (301 ‘A wooden block is floating on water kept in a beaker. 40% of the block is above the water surface Now the beaker is kept inside a lift that starts going upward with acceleration equal to g/2. The block will then (WB JEE 2014) (a) sink (b) float with 10% above the water surface (c) float with 40% above the water surface (d) float with 70% above the water surface A small metal sphere of radius a is falling with a velocity v through a vertical column of a viscous liquid. If the coefficient of viscosity of the liquid is 9, then the sphere encounters an opposing force of [WB JEE 2014) (a) 6mav &) ra (© 6rnav om A flow of liquid is streamline, if the Reynolds’ number is [CBSE AIPMT 2014) (a) less than 1000 (b) greater than 1000 (€) between 2000 to 3000 (a) between 4000 to 5000 A certain number of spherical drops of a liquid of radius r coalesce to form a single drop of radius R 7 sume VIET is the surface tension of the . quid, then [CBSE AIPMT 20 1) energy =avr( Lt a 77 Ris released eres -av7{ + 2)is bated (© energy -ovnf -4) is released (@) energy is if ther released nor absorbed op of some liquid of volume » 3. A drop " kame 0.04 em $8 the surface of a glass slide. Then, another glass Aide is placed om itm such a way that the hati forms a thin layer of area 20 cm? between the \yrfaces of the (wo slides. To separate the sides a flxce of 16%10° dyne has to be applied thesurfaces. The surface tension of the eae is placed fin dyne em") liquid is Dee (we Jee 2014 (60 70 (80 (@)90 : Under isothermal condition, energy E is su double the radius of the soap bubble. The value of E is (EAMCET 2014) i6rto (@n2a06 25. A 20 cm long capillary tube is dipped in water. The water rises upto 8 cm. If the entire arrangement is put in a freely falling elevator, the length of water. column in the capillary tube will be [UK PMT 2014) @4em —(b)20em_ (Bem —_(d) 10cm . The wettability of a surface by a liquid depends primarily on INEET 2013), 6) viscosity (b) surface tension of liquid and air (© density (d) angle of contact between the surface and the liquid 27. “The pressure exerted at any point in an enclosed fluid is transmitted equally in all directions”. This is ()24nro (@) Bxr?o known as D&KCET 2013) (a) Archimedes’ principle (b) Law of floatation (©) Pascal's law (4) Bernoulli's principle 28. There are two identical small holes on the opposite sides of a tank containing a liquid. The tank is open at the top. The difference in height between the two holes is h. As the liquid comes out of the two holes, the tank will experience a net horizontal force Proportional to UBKCET 2013} (a) Vr (bh (he? (ane 9. Water rises in a capillary tube upto a height of 10 cm Whereas mercury depresses in it by 3.42 em. If the angle of contact and density of mercury are 135° and of the surface 13.6 gce~! respectively, then the ratio of the . e neat tension of water and mercury will be Meaty Oo oo43) fo 16:5 2:13 (13:2) 5:16 Chapter 13: Fluid mechanics (38 80. Two capillary tubes of lengths in the ratio 2: 1 and radii in the ratio 1: 2 are connected in series. Assume the flow of the liquid through the tube is steady. Then, the ratio of pressure difference across the tubes is [EAMCET 2013] @1:8 — @ 16 ©) 32:1 Add ‘Two spherical soap bubbles of diameters 10 cm and 6 cm are formed, one at each end of a narrow horizontal glass tube. If the surface tension of the soap solution is 0.03 Nm", then the pressure difference in pascal between the two ends of the tube is (Kerala CEE 2013) (a) 16 ) 16 (0 0.016 (a) 0.08 (e) 0.16 The ratio of inertial force to viscous force of a fluid is called UKerala CEE 2013) (a) coefficient of viscosity (b) surface tension (6) Reynolds number (@) specific gravity (e) Lorentz number The excess pressure inside one soap bubble is three times that inside a second bubble. The ratio of the volume of first bubble to that of the second [MPPMT 2013) (1:27 @27— |@U9 — @ 91 ‘Water rises to a height of 20 mm in a capillary. If the radius of the capillary is made one-third of its previous value, then the new value of capillary rise will be (MPPMT 2013} 20 20 22 sam 6omm —(c) 22 mm mm oF o a? (@) 180 The unit of viscosity in the CGS system is poise (P) and that in SI is poiseuille (Pl). Which of the following statement is correct? (UP cPMT 2013) (@) 1P =1PL (®) 1 P=10PL (©) 10P=1PI (@) None of these - Liquid rises to a height of 2 cm in a capillary tube and the angle of contact between the solid and the liquid is zero. If the tube is depressed more now so that top of capillary is only 1 cm above the liquid, then the apparent angle of contact between the solid and the liquid is [UP CPMT 2013) @o ©) 30°) 60" (@) 90° The ratio of radii of two bubbles is 2:1 What is the ratio of excess pressures inside them? [UP CPMT 2012] 1:2 Oia (2st (4:1 A solid of density Dis floating in a liquid of density d. If Vis the volume of solid submerged in the liquid and V is the total volume of the solid, then V/V’ is eaual to > (UP ceMr 2012) we we @ 2 od D a np OD 640) OBJECTIVE PHYSICS Vol. 1 39. 10 min are taken to emptied a rectangular vessel of height A through an orifice in its bottom. How much time will it take to be emptied the vessel when half filled? [BCECE (Mains) 2012] (3 min WS min © Tamin (9 min 40. Water is moving with a speed of 5.18 ms”! through. a pipe with a cross-sectional area of 4.20 em?. The water gradually descends 9.66 m as the etoss-sectional area of pipe is increased to 7.60 em? ‘The speed of the flow of water at lower level is IBCEGE (Mains) 2012) (2.86 ms"? 3.0 ms" 3. (5.7 ms" 41, A block of ice in which a piece of stone is embedded is floating on water contained in a beaker. When all the ice melts the level of water in the beaker [BHU 2012) 0) rises (b) falls (©)remains unchanged —__(@) None of the above 42. Assertion The water rises higher in a capillary tube of small diameter than in the capillary tube of large diameter. Reason Height through which liquid rise in capillary tube inversely proportional to the capillary tube, [AIIMS 2012) (4) If both Assertion and Reason are true and Reason is the correct explanation of Assertion. (©) If both Assertion and Reason are true but Reason is not the correct explanation of Assertion, (6) If Assertion is true but Reason is false (@) If both Assertion and Reason are false. 43. Water rises in plant fibres due to (2) capillarty (©) fluid pressure (Manipal 2012) (0) viscosity (4) osmosis ‘44. The excess pressure inside a spherical soap bubble of radius 1 em is balanced by a column of oil (specific gravity = 0.8), 2 mm high, the surface tension of the bubble is (Manipal 2013] (@) 3.92 Nm“? (©) 0.0392 Nm! (©) 0.392 Now" (@) 0.00392 Nm“! 45, Water from a tap emerges vertically downward, initial velocity 4 ms“. The cross-sectional area gr the tap is A, The flow is steady and pressure jg constant throughout the stream of water. The distance h vertically below the tap, where the cross-sectional area of the stream becomes (2/3) 4, is © 7) (a) 2m 05m 46. The terminal speed attained by an aluminium. sphere of radius 1 mm falling through water at Oms 120) ) 1m ee) @4m 20°C will be close to [AMU 2012) @)9.2 ms (0) 6.9 ms (04.6 ms (a) 2.3 ms“ Assume laminar flow, specific gravity of Al = 2.7 and x104Pl) Mens 47, Water flows in a streamlined manner through a capillary tube of radius @ the pressure difference being p and the rate of flow Q. If the radius is reduced to a/2 and the pressure increased to 2p. The rate of flow becomes [AMU 2012) ae 3 48. A raindrop of radius 0.3 mm has a terminal velocity in air L ms“, The viscosity of air is 18 x 10~* poise. The viscous foree on itis (arMe 2012) (9) 10137% 10° dyne (101.73 10°? dyne (©) 1695x 10° dyme —_(@) 16.95% 10-4 dyne 49. If two soap bubbles of different radii are connected by a tube, then [Manipal 2012) (9) alr flow from bigger bubble to the smaller bubble tll Sires becomes equal (©) ar flow from bigger bubble to the smaller bubble tll sizes are interchanged (iw rm sales bbb to bigger (@) there is no flow of air — x point 13.1 PR wa 120) " int 13.2 oe 2 jot 133 ee eck point 13.4 Ve 2G) A Taking It together | 20 — mo 0) eo) oo 20 S052 aa 2® no Re ma a ma) 9208) we 102) me 120 m0) 22.6) pee wi) az whe) 152.06) Medical entrance special format questions Assertion and reason 1b) 2.0) Ma) Ze) Match the columns. 1tA+1,8-9q.C 90) 4A9g,.896099 2) Be 2a 3.0) 3.0) 3.0) 10) 23.10) 33. (0) 43. (b) 53.(0) 63.0) 73.(0) 83.(d) 93.(b) 103.3) 113.0) 123.(@) 133.) 143.0) 153.(0) 3.0) Bd) CMedical entrances’ gallery i) Ma a) 3.0) 4.0) 2.) 20) 20 32.0 42.00) 3.0 B10 23.10) 33.0) 43.0) 40 wie) 4.(6) 4d) 400) 4.00) 40) 210) 34.10) 44.10) 54.10) 64.) 74.18) B4.(d) 94.(0) 104.(b) 14.0 124.00) 134.00) 144.) 154.0) 4.(e) a0) +0) 21A448-+P.C 9) SA2a8 RCo ala 4.) 24.10) 34.10) “mo Answers 510) 5.) 5.0) 50 105-6) 15.0 125.(@) 135. (0) 145. (6) 155.1) 5.0) 15.0) 5.0 15.0 25.(0) 35.(0) 45.(0) 61>) 6.00) ee 6@ 6.) 16.10 26.(0) 36.(0) 46. (2) 56.(0) 66.(0) 76.10) 86.(0) 96. (0) 106.2) 116.) 126.(b) 136. (0) 146. (a) 156. (6) 6.0) 16.) 6.18) 16.6) 26.10) 36.0 46.10) 7.0 7.0) 70 7.0) 7.08) a7.4d) 27.14) 37.10) 47.0) 57.(0) 67.10) 77.12) 97.0) 97.13) 107, (0) Wn. 127.0) 7.0 147.) 157.0) 7.4) 17.8) 3.(A-sp.831C+p) 6 A>p.8 pC 99 7.00) 17.0) 27.10) 37.0) 47.0) a0 a0) 80 108. (>) 8.0) 128.4) 138.(0 148.) 8.) 18.) 8.10) 18.(0) 28.(b) 38.00) 43.00) 9.0) 9.18) 9.00) 9.0 19.18) 23.10) 39.0) 49.10) 59.10) 69.10) 79.10) 89.10) 99.(0) 108. (0) 119.) 129.0 139.8) 149.00) 9.1) 19.0) 9.10) 9.0) 29.) 39.10) 49.10) 10.10) 10.6) 10.18) 10.10) 10.10) 20.0) 30.0 40.(0) 50. (6) 60. (a) 70. (a) 80.(0) 90.0) 100. (6) 110. (0) 120.(2) 130.6) 140. (6) 150.(¢) 10.40) 10.(e) 20.(0) 30. (0) 40.(0) Hints and explanations Check point 13.1 1, (@ A fli comparises af both liquid anal ases, 2 (6) Dimensional formula for pressnte fy [ML AP?) 3. (8 Pointed ends have very small area therefore transits a lange pressure, “10x10 _ 7x10! ISN10-*N2 "30 NBN 10% mv? (©) Gauge pressure = absolute pressure ~ atmospheric pressure 4. (ed) Pressure, p= , x10" a 5. (8 Absolute pressure p= py + pyh Py = LOLN10° Pa P= 1.03510" kgm P= 101X10°Pa + 51,5N10°Pa = 1O1N10°Pa + 51.5 x10°Pa =52.5x10°Pa ere 7. (©) The shape of the container does not affect the pressure, The four vessels AR C and D have different shapes and hold different amount of water. The free surface of water in all vessels ate in same horizental plane. The pressure at the four points E, F, G and H lying in the same horizontal plane will be B. (0 p= PHA PSS _ + 2/9) Yrs 23S 9 (a) Ifthe pressure at the oll-water interface in the left-arm is then the pressure in the right arm at the level of the interface will also be p. In the left arm, P= Po +p.gl+d) wll in the right arm, P= Po + Pag! i) Equating (i) and (i) t 135mm a - = pe = ae 11000 kg = 900 kg Tee aemm+ Sm * ¥ 10, (b) It is due to Pascal's law. 11. (6) Here, em F\=F and F, =mg= 1400 x10N According to paseal's law dem and Fi 4) f= Fx, = taxi08 «(4 finfish X= eos : wlio" 4) => T= cong ny column > Tax / congo xscep 13, (6) Lot weight of boat = Wand weight of stone =w Assuming density of water = Igce™!) Volume of water displaced Initially = jw + W) Later, volume displaced -( + 2) (© = density of stone = Water level eomes down, 14, (¢) Lotvbe the volume of eavities andV the volume of solid iron, Then (2222) «cron? Tetx6 Farber, eres tn weigh = thu +P or 1.078 + v) x10" x9.8 ae conciess v=0.12m! the law of buoyancy, we have Veg = 0.6 Voy, for the part of body outside oil and Vag = 0.429, for the part of body outside water, hence we get So, we have, Check point 132 1, (@) In the streamline flow Vj = constant; = constant, V; = constant Kane, 2. (b) For 1000 < Re < 2000, flow is unsteady. 3. (a) By equa ion of continuity AV = constant AW =AV, + AV, SS AKB=0.8Ax6+ 350 > AA=24A+ SAV => 4= 2.44 Lv 16 > vats - 7g 7 llms 4. (B) Bernoulli's theorem is a rmechanteal energy # based upon conservation of —— SS ig Accontng (0 equation of continuity : Aw, = Awe 1Oxl= Suv, = vy = 2a! Now applying Bernoulli's equation we get, 200 , 1000 4 (b) As water falls from a tap, the velocity of water goes on \nereasing hence from the equation of continuity, the eross mtion of water-stream goes on decreasing. If Ay = area of seteeection of the mouth of the tap and vy is the velocity of Giter at this point, and A, v are corresponding quantities at depth fy then Avo = Av =Vivolume rate of flow) h= 10cm 7. (8 Given, = 29 v= (Pah = SPxTO xO = VE ms"! 8. (a Velocity of efflux = Y2gh, which is independent of density of liquid. 9. (t) The lifting of an aeroplane is based on Bernoulli's principle. When airplane moves through air, pressure Bifference is created due to higher speed of air in the region above the wings Ft 10,06 Range of water = 24THT=R = 2] _ Check point 13.3 1. Unit of nisNmv?s 2 (9 Viscosity of water decreases with rise of temperature, 3. (@) Rate of flow through a tube is given by V- = 4.) When the tubes are fitted in parle! vot whet hg tant ort Sr oa raui+nh* ao 5. (0) F = 6mmrv i.e.sv has the dimensions of [> oF 8. (d Terminal velocity, vr 7° en ( fy <4 on, (R72, Chapter 13 : Fluid mechanics 643 eo Veimity boomer constant ternal velo 3 Be eet te sa of smaller op 08 10 tat ee age lune we G~) Now terminal velocity radius of 9, (d) When the small sized spherical bods’s fall iscous liquid fire its velocity increases and the: Constant which is called terminal velocity 10, (a) Before entering the water the velocity of ball is J 29%. If her entering the water this velocity does not chang Se Sse held be equal to the terminal velocity. Therefore. ~2He=si0 ; Jigh = 28 2 Gx10™)"0* = 10°F 9.8 81 (9.8x10°) Check point 13.4 1. (@ Surface tension T=F /Iand tension is Nin“! or Jm™ N_ke-ms? _ kg. 2. (a) Force on each side = 271 (due to two surface). 3. (a) Surface tension = E = —2*107 21” 210x107 oun 5.(0 W=6rRr Wer? IFradius double, then work done will become four times. 6. (0) Radius of curvature, = “=a, ie.,a straight plane. 7. (8 Express pressure, Ap = ST = 4%2%25 "107 r 1x10 =20Nm?=20Pa sr a2) 10. (a) ha ZToee fr E88 constant 9 = hm hyorhy= Be ‘Substituting the values, hy = (2.019) 6.0cm a) OBJECTIVE PHYSICS Vol. 1 A) Taking it together 1, (6) Surface tensi is found mainly in liquids. 2. (c) Water droplets assumes minimum area due to surface tension and becomes spherical 3. (a) ha PESOS. If Gis less than 90%, then h will be positive. 4, (6) A wetting liquid has a concave meniscus. 5. (6) For a body immersed in liquid, when the weight of the body is less than the upthrust then the body will float partially immersed, 6. (0) As brush is withdrawn, water susface formed over bristle tends to take smallest area due to surface tension. 7. (a) Water proof surface coatings are made up of materials which increases angle of contact and water does not wets the surface, 8. (8) For small drops, force of surface tension predominates ‘gravitational force and so, they are more spherical 9. (d) Cohesive force of water is greater than adhesive force and hence water does not wets the surface. 10. (c) This happens due to viscosity. 11. (a) Soluble impurities increases the surface tension. 12. (b) Surface energy of combined drop will be lowered, so excess surface energy will raise the temperature of the drop 13, (c) Water proofing agents increases angle of contact. 14, (8) As we know for a streamline flow of a liquid velocity of each particle at a particular cros-section is constant, because Av = constant (law of continuity) between two cross-section of a tube of flow. 15. (c) In streamlined flow, velocity of all the particles arriving at 2 ‘common point remains same with time (both in magnitude and direction, So, the KE = 41m? =a constant for llth pariles as all fluid particles have identical mass also. 16. (0) Velocity of different layers of a flowing fluid decreases with distance from the axis of the tube. It is maximum at centre and minimum at walls. 17. (@) Ina streamline flow at any given point, the velocity of each passing fluid particles remains constant. If we consider a ‘cross-sectional area, then a point on the area cannot have different velocities at the same time, hence two streamlines of flow cannot eross each other. 1B. (0) F =a, Weare =3N 19. (a) Specific gravity or relative density! weight in aie 60 =8u3 ~ Change of weight in water ~ 20 20. (a) Neglecting gravitational head, difference in pressure energy = difference in kinetic energy Soy? of vaa[P «2/10 ao sonal ali = 20 ms" or V400 ms (Po — po) 21. (8) When iron block was floating it displaces ¥; volume of Oe gcd eae ee TEGRAIRE Bie Soyo rn ome an of water, this vlume will be greater than volume V, of ig? fay ere absinthe pond it dp She yal as own values ie Pe emer ees erie 2 cos @ «1 if Gis less than A will be more, fl 7 22. (b) h= 23. (4) h 2, so for les wil be more 24, (d) With acute angle, of contact, liquid will rise in capillary 25, (a) Surface tension decreases with temperature and becomes zero at boiling point. 26. (b) Cohesive force decreases. So angle of contact decreases, 27. (@) Work done, W = 8xR"T Bx x10? «2 x10"? = 16n x10 2B. (0) weight is balanced by force of surface tension. : w= On 29, (¢) Radius of curvature of common surface is, Reman = 30. (9 ap =47 31. (0 Since, Ap & VR 32. (0) 1 atmosphere = 1.01 x 10° N Poh LOI «10° ip 77769 m= 8 km 33. (@) 21

You might also like